BIOCHEMISTRY (Nutrition ) Flashcards

1
Q

Why do we use Vitamin C to treat Methemoglobinemia ?

A

-Methemoglobinemia is a condition where Fe2+ of hemoglobin is Fe3+ ,

therefore vitamin C will help absorb more Fe2+ and therefore lower the levels of methemoglobin and transform it into hemoglobin ++ ++

we need to refuce fe3 into Fe 2 to get absorbed and get Oxygen from tissues

How well did you know this?
1
Not at all
2
3
4
5
Perfectly
2
Q

if the patient had crohn disease , and had a terminal ileum resection , what vitamin he would not be able to absorb ?

A

Vitamin B12 (because cobalamin is absorbed by the ileum )

How well did you know this?
1
Not at all
2
3
4
5
Perfectly
3
Q

What are the water soluble vitamins ?

A
  • Vitamins B and Vitamin C

-Vit B1 , B2 , B3, B5 , B7 , B9, B12

-Vit C is named ascorbic acid (deficiency of vitamin C is called Scurvy +++ )

-water soluble vitamins get absorbed into the blood and excreted rapidly from the urine (so low toxicity rate par rapport with fat soluble vitamins )

How well did you know this?
1
Not at all
2
3
4
5
Perfectly
4
Q

What are fat soluble vitamins ?

A

Remember the mnemonic ‘‘DrAKE ‘’ the artist drake without the R , so fat soluble vitamins are : -vit D , -Vit A , Vit K , Vit E

How well did you know this?
1
Not at all
2
3
4
5
Perfectly
5
Q

gamma carboxylation needed for vitamin K takes place in :

A

the Liver +++

(warfarin inhibit gamma carboxylation +++ )

How well did you know this?
1
Not at all
2
3
4
5
Perfectly
6
Q

deficiency in vitamin K means that :

A

-I bleed a lot when injury

(prolonged PT and PTT while bleeding time is normal because the platelets are normal )

How well did you know this?
1
Not at all
2
3
4
5
Perfectly
7
Q

what’s the name of vitamin E ?

A

-Tocopherol or tocopheral of tocopherol acid

How well did you know this?
1
Not at all
2
3
4
5
Perfectly
8
Q

vitamin E excess can work like what ?

A

it can work like warfarin and inhibit gamma carboxylation

so whenever you give warfarin , pay attention to the intake of vitamin E in your patients , and of course , pay attention to the INR and the treatment to detect any abnormalities ,

because vit K deficiency is hemorrhage , ( patients can have serious hemorrhages like brain +++)

How well did you know this?
1
Not at all
2
3
4
5
Perfectly
9
Q

is vitamin A teratogenic ???

A

Yes , it is teratogenic

Note : for the treatment of acne we prescribe vitamin A , but before prescription , patient should be any kind of birth pills and B HCG should be negative ++++ ( MEDICO-LEGAL )

How well did you know this?
1
Not at all
2
3
4
5
Perfectly
10
Q

nutrients are divided into macronutrients and micrronutrients :

A

-Macronutrients : -carbohydrates , protein - fat

-Micronutrients : vitamins and minerals

How well did you know this?
1
Not at all
2
3
4
5
Perfectly
11
Q

overdose of minerals and its effect on the kidneys ?

A

if overdose on minerals —-) damage to the kidneys (tubules ) specially trace minerals like zinc …..(renal tubular acidosis ,

but if overdose of vitamins , a lot of times , the kidney can handle it

How well did you know this?
1
Not at all
2
3
4
5
Perfectly
12
Q

what is organic chemistry ?

A

-it is the chemistry of carbon (vitamins in general contains carbon )
minerals don’t contain carbon

if i take double dose of vitamins ,my kidney canhandle it for the most part , but for minerals nooo

How well did you know this?
1
Not at all
2
3
4
5
Perfectly
13
Q

zinc deficiency

A

-taste problems
-smell problems

How well did you know this?
1
Not at all
2
3
4
5
Perfectly
14
Q

an importance difference betweem vitamins and minerals is :

A

-The ability of kidney to handle them +++
vitamins are well handled by kidneys more than minerals which are toxic for kidneys (specially for the tubules )–) AKI —-) dialysis

How well did you know this?
1
Not at all
2
3
4
5
Perfectly
15
Q

The difference between cofactors and coenzymes ?

A

-coenzymes help enzymes : vitamins

-cofactor are minerals

How well did you know this?
1
Not at all
2
3
4
5
Perfectly
16
Q

Name 3 substances that determine your natural skin colour ?

A

-Melanine

-Beta caroténe

-Hemoglobin

How well did you know this?
1
Not at all
2
3
4
5
Perfectly
17
Q

vitamin A : (retinol , retinal , retinoid acid )

A

-anti oxydant
-rhodopsin for rhods of the retina ( night vision )

How well did you know this?
1
Not at all
2
3
4
5
Perfectly
18
Q

note of zinc and vitamin A

A

Zinc deficiency can give vitamin A deficiency +++ because zinc enter in the chemicals of forming vitamin A

How well did you know this?
1
Not at all
2
3
4
5
Perfectly
19
Q

What are the 2 medications that should never being prescribed in women without pregnancy test ?

A

-Methotrexate

-Vita A medications (example isotretinoine for acne vulgaris )

they are teratogenic ( MEDICOLEGAL+++ )
and always ask for a pregnancy test and ensure women are on contraceptive methods +++++++

How well did you know this?
1
Not at all
2
3
4
5
Perfectly
20
Q

What is the vitamin that treats cancer ( leukemia ) ???

vitamin can treat cancer ++++

A

-Vitamin A

(it is the treatment for acute promyelocyte leukemia , t(15,17 )

How well did you know this?
1
Not at all
2
3
4
5
Perfectly
21
Q

note about carotene and vit A

A

carotene is transformed to active vitamin A in the liver ++++

How well did you know this?
1
Not at all
2
3
4
5
Perfectly
22
Q

what is the rome of Rhods ? (Rhods = batonnets )

A

they are efficient for night vision

so if abnormal or defficient —-) Nyctalopia

and that what’s happened with vitamin A deficiency because vitamin A help make Rhodopsin +++

How well did you know this?
1
Not at all
2
3
4
5
Perfectly
23
Q

excess vitamin A :

A

it can give symptoms of intracranial pressure (pseudotumor cerebri )

How well did you know this?
1
Not at all
2
3
4
5
Perfectly
24
Q

vit A and vit E :

A

vitamin E can help absorb vitamin A

How well did you know this?
1
Not at all
2
3
4
5
Perfectly
25
Q

question : How can we differentiate between jaundice and hypercarotenemia CLINICALLY ???

A
How well did you know this?
1
Not at all
2
3
4
5
Perfectly
26
Q

what does vitamin D deficiency called in infants and adults (2 names for the same disease ) ?

A

-Infants : Rickets

-Adults : Osteomalacia

How well did you know this?
1
Not at all
2
3
4
5
Perfectly
27
Q

D2 and D3 names :

A

-D2: Ergocalciferol

-D3 : Cholecalciferol

How well did you know this?
1
Not at all
2
3
4
5
Perfectly
28
Q

breast milk is deficient in :

A

-Vit D
-Vit K
-Iron

How well did you know this?
1
Not at all
2
3
4
5
Perfectly
29
Q

Methemoglobinemia :

A

Met : change
Hemoglobin

emia : blood

it means the change of hemoglobin in the blood

vit C helps in reducing this form because vit C elevate the absorption of iron in its active Fe2+ form and then it can change Fe3 in the methemoglobin —-) help in treating methemoglobinemia ++++

How well did you know this?
1
Not at all
2
3
4
5
Perfectly
30
Q

Bacteria in the gut make vitamin K , we can have vitamin K also from the diet +++

A
How well did you know this?
1
Not at all
2
3
4
5
Perfectly
31
Q

a fact about vitamin K : (K1,K2, K3 )

A

K1 and K2 : natural fat soluble vitamins

K3 :synthetic water-soluble vitamin (naphthoquinone )—) that’s why we give it Intramuscular in birth and it get excreted rapidly by the kidneys +++++(neonates gut is sterile so they have no vitamin K at first ++++ )

How well did you know this?
1
Not at all
2
3
4
5
Perfectly
32
Q
A
How well did you know this?
1
Not at all
2
3
4
5
Perfectly
33
Q

What’s the major dietary lipids ??

A

1-Triglycerides (pancreas breaks them into glycerol +FFA )

2-Cholesterol

3-Essential fatty acids

4-Fat soluble vitamins (D,A,K,E)

if i have a malabsorption syndrome , i will not be able to absorb them

How well did you know this?
1
Not at all
2
3
4
5
Perfectly
34
Q

the following neural regions are most likely abnormal in this patient with B12 deficiency ++

A

High-Yield Summary
Animal products and fortified foods are the only dietary sources of vitamin B12, which is absorbed primarily in the ileum. A vegan diet and Crohn disease are both risk factors.

The neurologic symptoms of vitamin B12 deficiency are due to degeneration of dorsal columns, corticospinal tracts, and spinocerebellar tracts.

How well did you know this?
1
Not at all
2
3
4
5
Perfectly
35
Q

Vit B12 deficiency (usmle rx case correction )

A

This patient has multiple risk factors and signs of vitamin B12 deficiency. Vitamin B12 is naturally present in foods of animal origin, and fortified breakfast cereals and fortified nutritional yeasts are readily available sources of vitamin B12 that have high bioavailability. Vegans who specifically do not take supplements or fortified nutritional yeast are susceptible to deficiency. The patient also has Crohn disease, which commonly affects the terminal ileum, where vitamin B12-intrinsic factor is absorbed, making her very likely to be deficient in vitamin B12. Other possible risk factors for vitamin B12 deficiency include Helicobacter pylori or Diphyllobothrium latum infection, gastric bypass surgery, and poor dietary intake. The most prominent findings in vitamin B12 deficiency are glossitis (large, shiny tongue), macrocytic megaloblastic anemia with hypersegmented neutrophils, and neurologic findings including dementialike symptoms and ataxia.

Step 2: Disease Mechanism

The neurologic findings associated with vitamin B12 deficiency are due to the degeneration of neurons and myelin in the dorsal columns, lateral corticospinal tracts, and spinocerebellar tracts (see image).

How well did you know this?
1
Not at all
2
3
4
5
Perfectly
36
Q

A 57-year-old man is brought to the emergency department because of blurry vision, difficulty standing, and mental confusion. The patient’s companion says he is having difficulty remembering to do everyday tasks. On physical examination, the patient appears malnourished, and his gait is wide-based and unsteady. His abdomen is distended, and thin branching telangiectasias are noted. There is bilateral nystagmus on lateral gaze. A comprehensive metabolic panel shows:

Albumin: 1.8 g/dL
Total bilirubin: 3.2 mg/dL
Aspartate aminotransferase: 302 U/L
Alanine aminotransferase: 149 U/L
Alkaline phosphatase: 85 U/L

Which of the following neuropathologic findings are most consistent with this patient’s symptoms?

A

the symptomatology of this patient along with labs can lead us to think about :

-cirrhosis or hepatological situation that give a portal hypertension ( can explain the abdominal distension and telangiectasia en tete de méduse ) , also neurological findings in this patient can be related to hyper ammonemia ——) ammoniac is neuro toxic ++++

but what exactly the mechanism ? or where does ammonium deposit ? i don’t know —–) NOOOOT CORRECT FALSE UNDERSTANDING Of the clinical situation +++

Symmetric lesions in the mammillary bodies
Symmetric lesions in the mammillary bodies
63%
That’s Correct!
ScholarRxHigh-Yield Summary
Chronic alcohol dependency often leads to malnutrition and thiamine (vitamin B1) deficiency. This may eventually result in the clinical syndromes of Wernicke encephalopathy and Korsakoff psychosis.
Wernicke encephalopathy is associated with symmetric damage to the mammillary bodies.
Step 1: Disease Diagnosis

This patient presents with several signs of chronic alcohol dependency, including malnutrition, ataxic gait (wide and unsteady), distended abdomen (indicative of ascites), and spider angiomata (thin branching telangiectasias). The patient’s lab elevated aspartate aminotransferase and alanine aminotransferase levels are likely the result of hepatocyte damage due to chronic alcohol abuse. Low albumin may be seen in patients who are malnourished. Malnutrition in patients with chronic alcoholism can lead to deficiency of thiamine (vitamin B1) due to reduced intake and impaired liver storage and utilization. This can result in two clinical syndromes: Wernicke encephalopathy and Korsakoff psychosis. Wernicke encephalopathy manifests with the classic triad of confusion, ophthalmoplegia, and ataxia, which are all observed in this patient. Korsakoff psychosis (personality change, confabulation, and memory loss) is a late manifestation of Wernicke encephalopathy and is often irreversible.

Step 2: Disease Mechanism

Patients with Wernicke encephalopathy are commonly found to have symmetric lesions in the mammillary bodies.

The other choices are incorrect:
Atrophy of the caudate nucleus is seen in Huntington disease, which presents with chorea and dementia.
Depigmentation in the substantia nigra pars compacta is seen in Parkinson disease, which presents with bradykinesia.
Lesion of the subthalamic nucleus would cause hemiballismus, which presents with wild, uncontrolled flailing of the contralateral extremities.
Neurofibrillary tangles and widening of ventricles is characteristic of Alzheimer disease, which presents with progressive dementia.

How well did you know this?
1
Not at all
2
3
4
5
Perfectly
37
Q

case n3 :
A 72-year-old woman comes to the clinic because of a 3-month history of fatigue. She lives alone and, over the past 6 months, has been consuming a diet of mostly tea, bread, and cheese due to financial concerns. She does not drink alcohol or take any medications. On physical examination, there is conjunctival pallor. The rest of the physical examination discloses no abnormalities. Results of complete blood count show:

Hemoglobin: 8.7 g/dL
Hematocrit: 26%
Mean corpuscular volume: 113
Leukocytes: 7,000/mm3
Platelets: 140,000/mm3

A

Increased blood homocysteine
58%
That’s Correct!
ScholarRxHigh-Yield Summary
Folate deficiency causes macrocytic anemia.
Homocysteine levels are typically elevated in folate deficiency.
Step 1: Disease Diagnosis

This patient likely has folate deficiency, given her anemia (fatigue, pallor), high mean corpuscular volume (macrocytic anemia), and limited diet. This form of anemia can be due to folate or vitamin B12 deficiencies, liver disease, heavy alcohol use, hypothyroidism, and some antiretroviral drugs used for the treatment of HIV. In the absence of findings of liver disease (hepatomegaly, ascites, edema) or hypothyroidism (slow pulse, goiter), or the use of other medications, the most likely cause is a vitamin deficiency. This patient has a hematologic picture of megaloblastic anemia but lacks neurologic symptoms, suggesting a deficiency of folate rather than vitamin B12. Also, her limited diet includes cheese, which is rich in vitamin B12 but completely devoid of folate, suggesting folate deficiency as the most likely diagnosis. Unlike B12 deficiency, folate deficiency can present within months after removal of folate from the diet, whereas B12 deficiency takes years to develop because the body maintains B12 stores that are enough for about 3-5 years.

Step 2: Disease Mechanism

Folate deficiency results in increased homocysteine levels. This is because folate acts as a one-carbon carrier (5-methyl tetrahydrofolate [THF]) for methylation reactions, including the conversion of homocysteine to methionine. In this reaction, Vitamin B12 acts as a cofactor for the enzyme methionine synthase, and 5-methyl THF acts as the methyl donor. Folate deficiency halts this reaction and thus results in increased homocysteine levels. THF also donates a methyl group in the methylation of deoxyuridine monophosphate into deoxythymidine monophosphate, which is necessary for DNA synthesis. Folate deficiency thus results in reduced tetrahydrofolate and impaired DNA synthesis, which then halts replication without disturbing transcription, resulting in megaloblastic anemia (see figure).

How well did you know this?
1
Not at all
2
3
4
5
Perfectly
38
Q

note to know about B12 :

A

-Cheese is rich with vitamin B12

How well did you know this?
1
Not at all
2
3
4
5
Perfectly
39
Q

An 8-day-old boy is brought to the pediatrician’s office for a checkup. His mother reports that his gums bleed after breastfeeding, but he otherwise has been eating, stooling, and voiding well. He had an uncomplicated vaginal birth at 40 weeks at home. On examination, there is some mucosal bleeding on his gums and a small blood stain on the inside of his diaper around his umbilicus. Laboratory results show:

Prothrombin time: 40 sec
International normalized ratio: 2.1
Partial thromboplastin time: 55 sec

Which of the following is the most likely cause of this infant’s presentation?

A

Immature gut microbiome
50%
That’s Correct!
ScholarRxHigh-Yield Summary
A newborn, particularly after a home birth, presenting with mucosal, gastrointestinal, or intracranial bleeding along with elevated prothrombin time and partial thromboplastin time most likely has vitamin K deficiency. Newborns are typically given a supplemental injection of vitamin K in the hospital after birth.
Newborns are typically vitamin K deficient at birth due to immature gut microbiome, which is the primary source of vitamin K.
Step 1: Disease Diagnosis

This 8-day-old boy presents with bleeding in the setting of an increased prothrombin time, partial prothrombin time, and international normalized ratio. This presentation is most suggestive of vitamin K deficiency. Supplemental vitamin K injections are normally administered in hospital births to prevent hemorrhagic complications. Because this infant was born at home, he most likely did not receive vitamin K.

Step 2: Disease Mechanism

Vitamin K is produced by the gut microbiota. Because newborns have an immature gut microbiome, they do not produce adequate amounts of vitamin K. In addition, breast milk has low vitamin K content. Vitamin K is necessary for the production of factors X, IX, VII, and II and proteins C and S, which are important for the extrinsic and intrinsic pathways of the coagulation cascade. Deficiencies in these factors can be detected by increased prothrombin time and partial thromboplastin time, which assess the extrinsic and intrinsic pathways, respectively. Complications of vitamin K deficiency include intestinal, intracranial, and mucosal bleeding, which may lead to death. Because vitamin K (as well as A, D, and E) is fat-soluble, conditions resulting in decreased fat absorption are also risk factors for vitamin K deficiency.

How well did you know this?
1
Not at all
2
3
4
5
Perfectly
40
Q

case usmle RX :
A 3-day-old girl is brought to the emergency room by her mother after having a seizure. The patient was born at home at 37 weeks of gestation to a 28-year-old primigravida. The mother did not receive prenatal care. Physical examination reveals a small amount of dried blood around the umbilicus and areas of ecchymoses on the lower extremities. A cranial ultrasound shows blood in the right lateral ventricle. Laboratory studies are as follows:

Hemoglobin: 12.5 g/dL
Platelets: 200,000/mm3
Prothrombin time: 19 sec
Partial thromboplastin time: 47 sec
Bleeding time: 4 min
Stool guaiac test result: positive

Decreased action of which of the following underlies the patient’s most likely underlying condition?

A

γ-Carboxylation of glutamic acid

γ-Carboxylation of glutamic acid
62%
That’s Correct!
ScholarRxHigh-Yield Summary
A neonate who presents after a home birth with ecchymoses, intracranial hemorrhage, and bleeding from the umbilicus and gastrointestinal tract likely has vitamin K deficiency.
Vitamin K is necessary for the γ-carboxylation of glutamic acid residues on clotting factors II, VII, IX, and X and two anticoagulant glycoproteins, proteins C and S.
Step 1: Disease Diagnosis

This patient has a likely diagnosis of hemorrhagic disease of the newborn secondary to vitamin K deficiency, given the history of home birth, lack of prenatal care, and signs of increased bleeding (dried blood at umbilicus, ecchymoses, and positive stool guaiac test result suggesting gastrointestinal bleeding). The ultrasound demonstrates blood in the right lateral ventricle, which is consistent with intraventricular hemorrhage from germinal matrix bleeding (which caused the patient’s seizure). This diagnosis is further supported by the increased prothrombin time (PT) and partial thromboplastin time (PTT), with a normal bleeding time and normal platelet count. Because newborns do not have the intestinal flora that synthesizes most of the vitamin K in adults, infants born in U.S. hospitals are given an injection of this essential vitamin at birth.

Step 2: Disease Mechanism

Vitamin K is necessary for the γ-carboxylation of glutamic acid residues on clotting factors II, VII, IX, and X and two anticoagulant glycoproteins, proteins C and S. Because these clotting factors are part of both the intrinsic and extrinsic pathways of the coagulation cascade, vitamin K deficiency causes prolongation of PT and PTT.

The other choices are incorrect:

The following do not describe a coagulation defect:
Decreased activity of lysyl hydroxylase describes scurvy, in which the lack of vitamin C (a necessary cofactor for lysyl hydroxylase) results in weakened collagen that manifests with increased bruising, poor wound healing, and swollen gums.
LFA-1 integrin defects result in leukocyte adhesion deficiency that manifests with recurrent bacterial infections and delayed separation of the umbilical cord.
Interleukin-8 is a neutrophil chemoattractant, and defects would cause problems with the innate immune response.
The following would have different coagulation profiles:
Deficiency of GPIb-IX-V complex results in Bernard-Soulier disease. It would show thrombocytopenia, increased bleeding time, normal PT, and normal PTT.
A decrease in ADAMTS13, an enzyme that cleaves von Willebrand factor, results in thrombotic thrombocytopenic purpura. It usually manifests with a normal PT and PTT, along with a classic pentad of fever, microangiopathic hemolytic anemia, thrombocytopenia, renal dysfunction, and neurologic abnormalities.

How well did you know this?
1
Not at all
2
3
4
5
Perfectly
41
Q

Mnemonic to memorize fat-soluble vitamins :

A

-DrAKE without the r (D,A,K,E )

How well did you know this?
1
Not at all
2
3
4
5
Perfectly
42
Q

A 60-year-old woman with a history of type 2 diabetes mellitus, chronic kidney disease, and hypertension comes to the clinic for follow-up care. Examination shows ankle edema. Serum studies show:

Creatinine: 4.7 mg/dL
Glucose: 320 mg/dL
Ca2+: 6.5 mg/dL
Phosphate: 6.0 mg/dL (normal: 2.8-4.5 mg/dL)
Parathyroid hormone (PTH): 672 pg/mL (normal: 10-55 pg/mL)

Which of the following best explains the abnormal serum studies in this patient?

A

Chronic kidney disease
71%
That’s Correct!
High-Yield Summary
Secondary hyperparathyroidism is a major complication of chronic renal disease and presents with hypocalcemia and hyperphosphatemia.
Step 1: Disease Diagnosis

This patient likely has secondary hyperparathyroidism due to chronic kidney disease (CKD). This is based on her elevated PTH in the setting of hypocalcemia, hyperphosphatemia, and reduced renal function (high serum creatinine). The patient’s diabetes mellitus likely led to her CKD (diabetic nephropathy). A decreased glomerular filtration rate leads to increased serum creatinine and hyperphosphatemia (as both creatinine and phosphate are mainly filtered for excretion). The high phosphate reduces the circulating serum calcium by precipitation, leading to hypocalcemia. Lastly, the low calcium causes the parathyroid gland to oversecrete PTH, which stimulates bone reabsorption and over time causes renal bone disease (osteitis fibrosa cystica).

How well did you know this?
1
Not at all
2
3
4
5
Perfectly
43
Q

Homeostasis of Zinc :

A

-zinc is a mineral found in poultry , oysters , and fish

-absorbed in duodenum and jujenum

-Transferred via portal circulation to liver and stored in the liver

High yeald : Liver is the main organ involved in maintaining zinc homeostasis ++++

How well did you know this?
1
Not at all
2
3
4
5
Perfectly
44
Q

case usmle RX :
A 41-year-old woman reports months of worsening tingling in her feet. She has not traveled outside of New York City in the last 5 years and eats a balanced diet. She has no medical or surgical history. Physical examination reveals mild jaundice, and there is inflammation and a glazed, atrophic appearance of the tongue. Neurologic examination is significant for decreased vibration sense and decreased strength in the lower extremities. A peripheral blood smear is shown.

A

Autoantibodies against gastric epithelium
52%
That’s Correct!
ScholarRxHigh-Yield Summary
Vitamin B12 deficiency results in megaloblastic anemia, glossitis, and both motor and sensory symptoms (eg, paresthesia).
Pernicious anemia because of autoantibodies against gastric epithelium is the most common cause of vitamin B12 deficiency in patients without other risk factors.
Step 1: Disease Diagnosis

This patient has vitamin B12 deficiency, based on her paresthesias, glossitis, ineffective erythropoiesis causing hemolysis (with mild jaundice), and megaloblastic anemia with hypersegmented neutrophils on peripheral smear. Complete blood cell count would show anemia and high mean corpuscular volume. Causes include pernicious anemia, gastric atrophy, vegan diets, postgastrectomy malabsorption of vitamins, inflammatory bowel disease, pancreatic insufficiency, or tapeworm infection (in developing countries).

Step 2: Disease Mechanism

In patients without other risk factors, the most common cause of vitamin B12 deficiency is pernicious anemia, a disorder in which dietary vitamin B12 is not absorbed as a result of gastric parietal cell atrophy and the subsequent absence of intrinsic factor, a secreted protein required for vitamin B12 absorption. The disease occurs because of autoantibodies against gastric epithelium (parietal cells), and it is associated with other autoimmune disorders, such as thyroiditis.

How well did you know this?
1
Not at all
2
3
4
5
Perfectly
45
Q

Usmle Rx case :
A 4-year-old boy is brought to the pediatrician by his parents because of skin rashes that appear on sun-exposed areas and frequent episodes of diarrhea. On physical examination, the patient exhibits nystagmus, ataxia, and tremor. The child is noted to have delayed development. Urinalysis reveals higher-than-normal levels of certain amino acids and a deficiency of a water-soluble vitamin.

Abnormal metabolism or transport of which of the following amino acids characterizes the most likely diagnosis in this patient?

A

Tryptophan
56%
That’s Correct!
ScholarRxHigh-Yield Summary
Hartnup disease may present with photosensitive dermatitis, dementia, and diarrhea.
In Hartnup disease, the urinary and gastrointestinal systems poorly reabsorb the neutral amino acids due to a defective transporter. Loss of tryptophan leads to reduced niacin synthesis.
Step 1: Disease Diagnosis

A patient who presents at a young age with delayed development, an abnormal gait, tremor, nystagmus, sun-sensitive dermatitis, and diarrhea is likely to have Hartnup disease, a rare secondary form of pellagra (niacin deficiency). The symptoms of pellagra include dermatitis, dementia, and diarrhea; if untreated, it can result in death. The discoloration of the skin is similar to a sunburn and occurs on areas exposed to the sun.

Step 2: Disease Mechanism

Hartnup disease is caused by a defect in a specific transporter for neutral amino acids. This transporter is expressed in the renal tubules and gastrointestinal epithelium, and dysfunction results in decreased gastrointestinal absorption and decreased kidney reabsorption of neutral amino acids such as tryptophan. Tryptophan is a precursor for niacin synthesis in the liver. In patients with Hartnup disease, niacin deficiency occurs as a result of tryptophan deficiency. Other causes of niacin deficiency include malabsorption (eg, inflammatory bowel disease) and severe malnutrition, including patients with heavy alcohol use who may otherwise not eat balanced diets.

How well did you know this?
1
Not at all
2
3
4
5
Perfectly
46
Q

A 60-year-old woman comes to the physician because of chronic diarrhea and a skin rash that has been intensifying over the past 8 months. The rash worsens on sun exposure. The patient appears slightly confused and has trouble remembering what she did earlier in the day. Physical examination reveals the patient’s skin is discolored, as seen in the image.

Which of the following adverse effects is this patient most likely to experience if she is overtreated for her condition?

A

Facial flushing
57%
That’s Correct!
ScholarRxHigh-Yield Summary
The symptoms of niacin (vitamin B3) deficiency can be remembered with the four Ds: dermatitis, diarrhea, dementia, and death.
Excessive niacin supplementation can cause facial flushing, which can be prevented by pretreatment with aspirin.
Step 1: Disease Diagnosis

This patient presents with symptoms most consistent with pellagra, which occurs as a result of niacin (vitamin B3) deficiency. These symptoms include diarrhea, confusion, and an erythematous maculopapular rash on sun-exposed areas. The 4 D’s of niacin (vitamin B3) deficiency are Dermatitis, Diarrhea, Dementia, and Death.

Step 2: Drug Adverse Effects

Pellagra is treated with niacin (vitamin B3). Excessive niacin supplementation can cause facial flushing due to release of prostaglandins from dermal Langerhans cells, resulting in vasodilation. Facial flushing can be avoided if the patient is pretreated with antiprostaglandins, such as aspirin.

How well did you know this?
1
Not at all
2
3
4
5
Perfectly
47
Q

usmle rx case :
A 43-year-old woman visits her obstetrician for her scheduled checkup at 20 weeks of gestation. She has a history of hypertension and acne, both controlled with medication. Her obstetrician recommends that she have a series of screening tests. Her human chorionic gonadotropin, estriol, and inhibin levels are within normal limits, but her α-fetoprotein level is elevated.

Which of the following is the most likely cause of this patient’s abnormal laboratory value?

A

Inadequate folic acid intake
66%
That’s Correct!
ScholarRxHigh-Yield Summary
Elevated AFP levels in the amniotic fluid and maternal serum can be a sign of neural tube closure defects.
Folate deficiency can lead to neural tube defects.
Step 1: Disease Diagnosis

Screening tests in this patient reveal normal human chorionic gonadotropin, estriol, and inhibin levels but increased α-fetoprotein (AFP). An elevated AFP level in amniotic fluid and maternal serum may indicate a neural tube defect. Neural tube defects are caused by the failure of the neural tube to close (fourth week). The AFP level is elevated because it is leaking into the amniotic fluid from the spinal canal.

Step 2: Disease Risk Factors

Folic acid deficiency during pregnancy has been implicated in causing neural tube defects. Therefore inadequate folic acid intake is a possible cause of this patient’s elevated AFP level.

How well did you know this?
1
Not at all
2
3
4
5
Perfectly
48
Q

Vitamin B1 deficiency : ( thiamine deficiency )

A

-Beri beri (dry and wet beri beri )

due to a diet rich in white rice ++++

normally brown rice is nourished with B1 , but if we manifactured and transformed brown to white rice we will have deficiency in B1 ++++

How well did you know this?
1
Not at all
2
3
4
5
Perfectly
49
Q

beri beri ( dry and wet musical mnemonic )

A

-Dry : neuropathy

-wet : dilated cardiomyopathy

and let’s sing this babyy +++++

How well did you know this?
1
Not at all
2
3
4
5
Perfectly
50
Q

The best source of energy to the brain is :

A

-Glucose(come from carbohydrate )

How well did you know this?
1
Not at all
2
3
4
5
Perfectly
51
Q

A 43-year-old man comes to the clinic because of a painful rash on his forearms and neck, muscle weakness, and loose stools. He has a history of alcohol use disorder. On physical examination, the patient is agitated and disoriented. His tongue is enlarged and erythematous, and the affected skin appears thickened and hyperpigmented.

A deficiency in which of the following characterizes the most likely diagnosis in this patient?

A

Tryptophan
35%
That’s Correct!
ScholarRxHigh-Yield Summary
Patients with pellagra classically present with the triad of diarrhea, dementia, and dermatitis.
Pellagra can occur as a result of tryptophan or niacin deficiency.
Step 1: Disease Diagnosis

This patient most likely has niacin (vitamin B3) deficiency, also called “pellagra,” based on his presentation with a painful upper extremity rash, diarrhea, muscle weakness, enlarged and erythematous tongue, mental status changes, and history of alcohol use disorder (AUD). Individuals who are homeless or affected by AUD are at risk for developing dietary vitamin deficiencies. The symptoms of pellagra are classically described as the 3 Ds: Diarrhea, Dementia, and Dermatitis. Glossitis is another important symptom. Pellagra is an Italian word meaning “thickened skin,” which describes the hyperpigmentation on sun-exposed areas of the body (as seen in the image). The rash’s circumferential distribution in the C3/C4 dermatome is known as the “Casal necklace.”

Step 2: Disease Mechanism

Because niacin is derived from tryptophan, a decrease in tryptophan absorption or an increase in tryptophan metabolism results in pellagra. Niacin is an essential vitamin due to its roles as the building blocks in the production of NAD and NADP, which are cofactors that serve as oxidizing and reducing agents. Because these cofactors are required for metabolism of glucose and other metabolites, tissues throughout the body are affected. Primary pellagra results from a dietary deficiency in niacin or tryptophan itself, while secondary pellagra results from a preexisting condition that results in deficiency, such as anorexia nervosa, Crohn disease, or carcinoid syndrome.

How well did you know this?
1
Not at all
2
3
4
5
Perfectly
52
Q

A 35-year-old woman presents with fever, diarrhea, and right-lower-quadrant abdominal pain. She has had similar symptoms intermittently for 10 years. Pulse is 105/min, and respirations are 22/min. Physical examination reveals 4/5 strength bilaterally in the upper and lower extremities and diminished vibratory sensation in the feet. Laboratory tests show a WBC count of 12,000/mm3, a hemoglobin concentration of 9 g/dL, and a mean corpuscular volume of 112 fL.

Which of the following is most likely to be found on further workup?

A

Elevated methylmalonic acid level
66%
That’s Correct!
ScholarRxHigh-Yield Summary
Crohn disease may present with intermittent chronic abdominal pain and diarrhea and often affects the terminal ileum.
Vitamin B12 deficiency, a cause of megaloblastic anemia, is associated with a number of clinical and laboratory abnormalities, including fatigue and peripheral neuropathy.
Vitamin B12 deficiency is characterized by elevated levels of methylmalonic acid and homocysteine.
Step 1: Disease Diagnosis: Crohn Disease

The patient’s presentation consists of intermittent abdominal pain, diarrhea, and fever, suggestive of Crohn disease. Crohn disease often involves inflammation of the terminal ileum.

Step 2: Disease Diagnosis: Megaloblastic Anemia

The terminal ileum is the site of vitamin B12 absorption, and chronic terminal ileitis may lead to megaloblastic anemia (elevated mean corpuscular volume) resulting from vitamin B12 deficiency. Vitamin B12 deficiency is associated with a number of clinical and laboratory abnormalities, including fatigue and peripheral neuropathy presenting (as in this case) with tingling and numbness in the hands or feet, along with diminished vibratory sense.

Step 3: Disease Diagnosis: Methylmalonic Acid Level

Elevated levels of methylmalonic acid and homocysteine may be seen with vitamin B12 deficiency. Vitamin B12 is a coenzyme for methylmalonyl-coenzyme A (CoA) mutase, the enzyme that converts methylmalonyl-CoA to succinyl-CoA. A deficiency in vitamin B12 will subsequently result in an elevated methylmalonic acid level. Vitamin B12 is also a coenzyme for the enzyme methyltransferase, which is responsible for the formation of methionine from homocysteine.

The other choices are incorrect.
The following are associated with anemia but not macrocytic anemia:
Markedly hypocellular bone marrow is characteristic of aplastic anemia, characterized by pancytopenia and typically a normocytic anemia.
Increase in hemoglobin A2 level is seen in β-thalassemia minor, which manifests with a hypochromic microcytic anemia.
A positive direct Coombs test is seen in warm antibody autoimmune hemolytic anemia, which is mediated by IgG autoantibodies that react with RBC surface antigens.
The following choice does not fit this patient’s presentation:
The patient does not have a clinical history indicative of dilated cardiomyopathy, which would present with fatigue, dyspnea on exertion, orthopnea, increasing edema, tachypnea, and tachycardia.
The remaining choice is the opposite of the expected laboratory value:
Vitamin B12 deficiency does not lead to a low homocysteine level but to an elevated homocysteine level.

How well did you know this?
1
Not at all
2
3
4
5
Perfectly
53
Q

2 causes of edema of nutritional origin ?

A

-Diminished protein intake (malnutrition , kwarshiorkor )

-Diminished vitamin B1 intake (wet beriberi )

How well did you know this?
1
Not at all
2
3
4
5
Perfectly
54
Q

Thiamine first , glucose second ++++

A

wernicke korsakoff syndrome

How well did you know this?
1
Not at all
2
3
4
5
Perfectly
55
Q

A 71-year-old woman is brought to the emergency department by her daughter because of fatigue, dizziness, and palpitations. For the past few months, her diet has consisted of small quantities of coffee, rice crackers, and jelly. Laboratory studies show hemoglobin of 10.9 g/dL, mean corpuscular volume of 105.9 fL, elevated homocysteine, and normal methylmalonic acid.

A defect in which mechanism characterizes the most likely diagnosis in this patient?

A

Transfer of methyl groups
53%
That’s Correct!
ScholarRxHigh-Yield Summary
Macrocytic anemia is characterized by generalized symptoms of anemia, low hemoglobin, and increased MCV.
Megaloblastic anemia, a type of macrocytic anemia, is often caused by folate deficiency, which is due to decreased intake of folate and can manifest within months.
The reduced form of folate functions as a cofactor in the transfer of methyl groups to homocysteine.
Step 1: Disease Diagnosis: Megaloblastic Anemia

The patient likely has megaloblastic anemia, due to the history of malnutrition; symptoms of fatigue, dizziness, and palpitations; and laboratory values demonstrating low hemoglobin, increased mean corpuscular volume (MCV), and increased homocysteine. A major classification of anemia is based on the RBC size: macrocytic (MCV >100 fL), normocytic (MCV 80-100 fL), or microcytic (MCV <80 fL). Because this patient’s RBCs show an MCV of 105.9, she has macrocytic anemia, and because homocysteine is elevated, it is specifically a megaloblastic anemia.

Step 2: Disease Diagnosis: Folate Deficiency

Megaloblastic anemia has two principal causes: vitamin B12 (cobalamin) deficiency and folate deficiency. This patient’s diet over the last few months lacks fruit, vegetables, meat, fish, and dairy, so her megaloblastic anemia is most likely due to vitamin deficiency. Compared with vitamin B12, body stores of folate are minimal and can be depleted within a few months if intake is inadequate. Leafy green vegetables are a natural source of folate. In vitamin-B12-deficient anemia, both homocysteine and methylmalonic acid levels are increased. In addition, vitamin B12 deficiency presents with neurologic symptoms such as weakness and peripheral neuropathy.

Step 3: Normal Structure/Function

The tetrahydrofolate form of folate (folic acid) functions as a cofactor in the transfer of methyl groups during DNA and RNA synthesis, specifically purine synthesis. Tetrahydrofolate supplies methyl groups that are used in the conversion of homocysteine to methionine.

The other choices are incorrect:
Glutathione synthesis is defective in glucose-6-phosphate dehydrogenase deficiency and leads to a normocytic hemolytic anemia.
A defect in heme synthesis is often due to a lack of iron, and its deficiency can lead to microcytic anemia.
Cobalamin (vitamin B12) is required for isomerization of methylmalonyl-CoA. In addition to a macrocytic anemia, it would have an increased methylmalonic acid.
Defects in the synthesis of the spectrin-actin cytoskeleton, such as in hereditary spherocytosis, result in a hemolytic normocytic anemia.
Problems with the synthesis of β-globin chains can be caused by either β-thalassemia or sickle cell anemia. The anemia observed in β-thalassemia is microcytic, whereas that in sickle cell anemia is normocytic.

How well did you know this?
1
Not at all
2
3
4
5
Perfectly
56
Q

In the late 1990s, in an attempt to reduce the incidence of a specific category of birth defects, the US Food and Drug Administration mandated the supplementation of grain products with a coenzyme essential for nucleotide synthesis. This coenzyme is absorbed in the jejunum.

This chemical exerts its greatest effect on which of the following regions of the developing embryo?

A

Neural folds
62%
That’s Correct!
ScholarRxHigh-Yield Summary
Folic acid is a coenzyme for 1-carbon transfers giving rise to nucleotides.
Maternal deficiencies of folate may lead to neural tube defects, including anencephaly or spina bifida.
Step 1: Pharmacotherapy

The chemical is folic acid. Folate is absorbed by the jejunum and is a key cofactor for nucleotide synthesis because it is a 1-carbon donor as part of the synthesis of deoxythymidine monophosphate (d-TMP) and deoxyuridine monophosphate (d-UMP) using thymidylate synthetase. The US Food and Drug Administration mandated in 1998 that cereal grain products be fortified with folic acid to decrease neural tube defects (NTDs) because only one in three pregnant women was consuming enough folic acid in her diet at that time. Folate supplementation should be started prior to conception because closure of the neural folds occurs very early in pregnancy, roughly around 4 weeks.

Step 2: Disease Mechanism

NTDs are the result of a failure of the neural folds to close (see image), which can result from insufficient folic acid intake. The neural plate folds to become the neural tube, which has openings at either end known as the neuropores. Failure of the neuropore to close will result in NTD. However, even if all neuropores properly close, NTD may result from other abnormal embryogenic processes. Failure of the anterior neural folds to close leads to anencephaly, whereas failure of the posterior neural folds to close leads to various types of spina bifida, including occulta, meningocele, and meningomyelocele.

The other choices are incorrect as development of the midgut, notochord, metanephros, and mesonephric tubules are not affected by maternal folate deficiency.

How well did you know this?
1
Not at all
2
3
4
5
Perfectly
57
Q

what’s the relation between measles and Vitamin A deficiency ?

A

In developing countries where vitamin deficiencies are common, infections like measles can consume vitamins and worsen the symptoms of vitamin A deficiency, the most common cause of preventable childhood blindness worldwide. The vitamin A deficiency caused the girl’s keratomalacia (softening of the cornea) shown in the photograph. Vitamin A deficiency can also lead to night blindness, dry eyes and conjunctivae (keratoconjunctivitis sicca, which may lead to stinging, burning, and blurry vision), and gray plaques. Prolonged cases can result in development of corneal ulceration and necrosis, leading to perforation and blindness.

How well did you know this?
1
Not at all
2
3
4
5
Perfectly
58
Q

what’s the components of Fat molecule ?

A

a fat molecule is normally composed by :-Glycerol + Fatty acid

so when they enter to our body , they get cut to glycerol and fatty acids

if the combination (carbon ) happens once –) monoglyceride

if the combination happens twice —) Diglyceride

if the combination happens 3 times —) Triglyceride

How well did you know this?
1
Not at all
2
3
4
5
Perfectly
59
Q

Fatty acids :

A

there is :
-Saturated Fatty acids (no double bonds ) —) saturated with hydrogen bonds , packs well together , they are usually solid at room temperature .

-Unsaturated Fatty acids (double bonds ) ——) liquid at room temperature ( monounsaturated or polyunsaturated fatty acids )—-) Omega 3 , Omega 6 or Omega 9

How well did you know this?
1
Not at all
2
3
4
5
Perfectly
60
Q

Omega 3 :

A

-linolenic acid ++++

-usually polyunsaturated

-alpha-linolenic acid , marine sources (DHA, EPA )

-the marine sources (DHA, EPA ) are found in anchoves , sardines +++

61
Q

Omega 6

A

-Linoleic acid , arachidonic acid ++

linoleic acid can only be obtained in the diet —) essential fatty acids

62
Q

Omega 9

A

-Oleic acid
-can be made by our body (therefore not essential)

63
Q

Cis and trans configuration :

A

-Cis : when the functional sites are in the same site with the double carbon bond

-Trans : when the functional sites are in the opposite site withe the double carbon bond,

Trans fats are associated with coronary heart disease +++

64
Q

chylomicron is a

A

lipoprotein (it has the outer membrane of phospholipids and protein , and a hydrophobic core of TG, cholesterol, and fat soluble vitamins DrAKE )

65
Q

The impact of fat (unsaturated fats ) on health :

A

-Polyunsaturated fats are :
_precursors to hormones like prostaglandins ,—-) stimulates endothelial cells to secrete nitric oxide —–) vasodilator —) decreases resistance to blood flow ——-) lowers Blood pressure .

  _Reduce total and LDL cholesterol ---) lower heart disease 

 _Long chains Omega 3s (DHA, EPA ) can lower plasma triglycerides ++ , DHA is important for the development of eyes and brain of younf infants

-

66
Q

Are saturated fats healthy ?

A

-Saturated fats increase cholesterol levels by increasing overall LDL , increase HDL , and increase LDL particle size

-Americal health association (AHA ) recommended limiting saturated fats (5 or 6 ° intake daily )

LIMIT SATURATED FATS INTAKE ++++

-A lot of researchs linked saturated fats to heart disease +++ and other researchs claim it is not true +++ —–) so it is always about MODERATION +++

67
Q

Unsaturated fatty acids :

A

there is : Cis and Trans fats +++

_Trans are solid at room temperature

_Cis are liquid at room temperature

68
Q

polyunsaturated fatty acids (omega 3 , 6 ) are essential

A

our body can’t make them , we should bring them from the diet

69
Q

Omega 3 ,and Omega 6 effects :

A

-Omega 6 : can have inflammatory effects

-Omega 3 : can have antiinflammatory effects

the ratio of omega 6 and omega 3 can help decrease heart risk and inflammatory diseases

70
Q

Fats

A

-form a major component of our diet

-Fats are a major source of energy and a critical component of cells and tissues .

-fats help absorb vitamins (example : fat soluble vitamins are absorbed through lipase and biliary liquid ++ )

-fats can be converted to other molecules like prostaglandins

71
Q

Fats (biochemistry molecule )

A

-are composed of : Glycerol + Fatty acid

if one carbon chain between glycerol and fatty acid —-) Monoglyceride

if 2 —-) Diglyceride

if 3 —) Triglyceride

when a fatty acid is only composed of single bonds between carbon molecules —–) Saturated fatty acids ++++

when a fatty acid is composed of double bond carbon —-) Unsaturated fatty acids ++++

72
Q

the state of saturated and unsaturated fats at room temperature

A

-saturated fats are solid at room temperature

-Unsaturated fats are liquid at room temperature

73
Q

Omega 3 :

A

-usually polyunsaturated fatty acid (more than 2 double bonds )

-linolenic acid:
_ALA (Alpha linolenic acid)
_EPA (Eicosapentaeonic acid )
_DHA ( Docosahexaenoic acid )

EPA and DHA comes from marine sources (fish , anchoves , sardines )

74
Q

Omega 6 :

A

-Linoleic acid and arachidonic acid

-Usually polyunsaturated fatty acid

-linoleic acid is essential fatty acid (it can be obtained only in the diet )

75
Q

Omega 9 :

A

-oleic acid

-monounsaturated fatty acid (not essential fatty acid , can be made in our body )

76
Q

Cis and trans configuration

A
77
Q

Digestion of fat :

A

all foods are made up of a blend of fatty acids

if triglycerides enter the digestive tract , it got degraded by lipases into free fatty acids and glycerol

-bile salts degrades triglycerides into small molecules , then they got degraded by lipase ,

once triglycerides are degraded into FFA and monoglyceride they got into the micells into the intestines to get absorbed +++

-Inside the enterocytes , the micells release FFA and monoglycerides to get packed into TG and packed into CHYLOMICRONS (lipoprotein ) ++++

-The chylomicrons leaves the enterocytes in the lymphatic system (because it is too large to get into the endothelial cells of blood vessels )) , then dumped into the bloood

-once in the blood , the chylomicron releases FFA and monoglyceride in muscle , adipose tissues

78
Q

consumption of fats

A

it is not about the amount of fat we eat , but also the type of fat we eat (trans fats and saturated fats should be limited , consumption moderate )

79
Q

case osmosis about fats and lipids digestion :
A study is done on the processes of fat digestion and absorption in order to develop novel pharmacologics that can help prevent diseases associated with increased levels of fat and lipids in the blood. Which of the following is true regarding the digestion and absorption of fats and lipids?

A

answer:Absorbed fat and lipids get transported from enterocytes through lymphatic ducts

Major takeaway
Lipase breaks down fat globules into free fatty acids and monoglycerides, which are then absorbed by the enterocytes. Once inside enterocytes, the free fatty acids assemble again into triglycerides which are carried by chylomicrons into the lymphatic system.
Main explanation
Fats and lipids are essential components of the human diet, and the process of digestion and absorption of fats and lipids is a complex process that starts in the mouth.

The enzyme lipase is the major enzyme responsible for breaking down fats. It is secreted firstly in the saliva and starts the process of breaking down fat while in the mouth, which continues in the stomach too. However, the majority of the process of fat digestion and absorption takes place in the small intestines.

When fat globules reach the duodenum, pancreatic secretions containing lipase start breaking down fat globules. This process is facilitated by bile salts, which increases the surface area of fat globules for lipase to work. Lipase breaks down fat globules into free fatty acids and monoglycerides, which are then absorbed by the enterocytes.

80
Q

PUFA or polyunsaturated fatty acids are essential fatty acids

A

because they are not made in our body ,
we need to have them in the diet +++

-Decrease thre risk of cardiovascular disease

-Omega 6 and omega 3

81
Q

Vitamin D is found into 2 types (D2 and D3 )

A

-D2 =Ergocalciferol

-D3=

D2 is obtained through the diet

D3 is obtained through sun exposure

normally D2and D3 get transformed into CCF into the circulation , then they went into HYDROXYLATION ++++

-The first hydroxylation in the liver :
_CCF is transformed into 25-hydroxyCCF (it means that hydroxylation in carbon 25 )

-The second hydroxylation is in the kidneys :
_25 hydroCCF is transformed into 1,25 dihydroxy CCF (ACTIVE FORM OF VIT D )

-vitamin D in our body get 2 hydroxyation , the first in the liver and the second in the kidneys and then they got activated , we have active vitamin D (active form ++++)

82
Q

Total blood calcium :

A

8,5-10 mg /dl

83
Q

The gamma carboxylation of vitamin K happens in the :

A

LIVER++++

84
Q

water soluble vitamins vs fat soluble vitamins :

A

-water-soluble vitamins can flow in the blood without a transporter , so they are not stored in the body (except for vitamin B9 , and vitamin B12 )

-fat soluble vitamins get absorbed and stored in the body , so their toxicity is more than water soluble vitamins

85
Q

Water soluble vitamins are :

A

-Vitamin B

-Vitamin C

they are small and float freely in the blood (water soluble ) , so they are excreted in urine , —) toxicity rare +++

not like fat soluble vitamins which get stored in the adipocytes ++

86
Q

-Vitamin B numbers in biochemistry :

A

-B1, B2 ,B3
then skip 1

-B5,B6 ,B7
then skip 1

-B9 , B12

87
Q
A
88
Q

names of vitamin B s :

A

-B1 : Thiamin
-B2 : Rhiboflavin
-B3 :Niacin
skip 1

B5: Panthotenic acid
B6: Pyridoxine
B7 : Biotin
skip 1

B9 : folate
B12 : Cobalamin

89
Q

Wernicke encephalopathy :

A

-It is an imminently treatable disorder which if untreated can result in neurological permanent impairment .

-It is relatively common in patients with severe alcohol use disorder and is often underdiagnosed

-this condition is commonly missed because we think of the classic triad : -mental status changes -ocular test motility -gait abnormality (This triad only occurs in 10° of patients with wernicke encephalopathy ++++)

-if this condition is untreated —-) patients can develop korsakoff syndrome which is permanent (result in memory impairment )

–Diagnosis : clinical +++

–the main stay of treatment is the administration of THIAMIN +++ (vitamin B1 ) ;

-The MRI of the brain in wernicke encephalopathy : -Degeneration of mamillary body (responsible for memory ++)

-The treatment of Wernicke encephalopathy is to consider it as a : MEDICAL EMERGENCY +++(adress it immediately to prevent the development of korsakoff syndrome )

_Administer IV thiamine (guidelines : 500mg IV of B1 (2 times a day for 2-3 days )) then 250 mg daily for 2-3 days , then orally

90
Q

Role of vitamin B1 (thiamin ) :

A

-vitamin B1 is used for energy metabolism to support :
_growth
_development
_cellular functioning
of brain and nervous system ++++

-Thiamin deficiency includes pathologies like :
_Wernicke encephalopathy (acute and reversible )
_Korsakoff syndrome(chronic and irreversible )

91
Q

Wernicke korsakoff syndrome :

A

-Combination of wernicke encephalopathy + korsakoff syndrome

-Damage to the mamillary bodies and dorsomedial nuclei of the thalamus

92
Q

Beri beri

A
  • situation that is caused by Vitamin B1 deficiency

-we have :-dry and wet beri beri

93
Q

Vitamin B3 :

A

-Niacin or Nicotinic acid

-it can be synthesised in the liver from AA tryptophan
with the help of vitamin B2 and vitamin B6 , also we can have it from the dietary intake like all other vitamins

-It decreases LDL cholesterol , increases HDL cholesterol —–) Niacin can be used as a lipid lowering agent to treat dyslipidemia ++ (cholesterol )

vitamin B3 deficiency : Hartnup disease +++
_It is an autosomal recessive disease , deficiency of transporters (tryptophan ) in the GI tract and proximal convoluting tubules , —-) diminished tryptophan absorption ——-) not enough left to synthesize niacin ++++

-Carcinoid syndrome ( cancerous neuroendocrine cells start producing hormones ex serotonin , and serotonin is produced from tryptophan so tryptophan is decreased so there is not enough tryptophan left to synthesize B3 —–) B3 deficiency ++++

94
Q

B3 or niacin deficiency is manifested as :

A

Pellagra : 3D
_Diarrhea
_ Dementia
_ Dermatitis ( usually C3 , C4 dermatomes , in a broad collar shape like a casal necklace )

95
Q

Vitamin B5 name :

A

-Panthotenic acid

96
Q

Vitamin B6 : Pyridoxine

A

-water soluble vitamin

-symptoms : _neurological problems (seizures , hyperirritability , and peripheral neuropathy )

  • in the exam , they usually talk about patient who is under isoniazid (antituberculosis treatment ) , they say that he experienced peripheral neuropathy made of tingling or other )
97
Q

Ascorbic acid (note )

A

-Vitamin C is a cofactor in the hydroxylation of proline and lysine residues and is important in the synthesis of collagen , deficiency (scurvy ) is characterized by microvascular bleeding , gingivitis , and impaired wound healing .

98
Q

Uworld case conclusion about vitamin B9 (folate deficiency )

A

-In the form of tetrahydrofolate , folic acid participates in a number of single carbon transfer reactions , including the synthesis of nitrogenous bases like thymidine and uridine as well as several amino acids ,

-Deficiency can lead to megaloblastic anemia ++

-This patient’s macrocytic megaloblastic anemia without neurologic symptoms is most likely caused by folic acid deficiency.
Folic acid plays a methylationkey role as a coenzyme for single-carbon transfer as seen in methylation reactions and is essential for the biosynthesis of purines and the
pyrimidine thymidine. Deficiency of the vitamin is characterized by macrocytic megaloblastic anemia. The anemia is a result of diminished
DNA synthesis in erythropoietic stem cells, leading to cont inued cell growth without progression to mit osis, t hus present ing as macrocytosis.
Large ce lls are seen with mean ce ll volumes of> 100 1Jm3 and reduced levels of hemoglobin. Hypersegmented neut rophils (nuclei with six or
more lobes), as seen in the blood smear in the vignette image, ar e a dist inguishing featu re of megaloblastic anemia, caused by folic acid and
B12 deficiency. Folic acid is a water-soluble vitamin stored in smalll amounts by the body; thus a continuous supply is needed f rom foods such
as green, leafy vegetables, lima beans, and whole-grain ce reals. Folate deficiency is usually seen in pregnant women and alcoholics and is the
most common vitamin deficiency in the United Stat es. Folic acid supplementat ion by pregnant women reduces t he incidence of neural tube
defects .

99
Q

Thiamine deficiency-related diseases :

A

-werniche-korsakoff syndrome (confusion , ataxia , oculomotor abnormalities , and permanent memory deficits )

-Infantile and adult beri beri

Manifestations of infantile beriberi appear at age 2- 3 months and include a fulminant cardiac syndrome with cardiomegaly, tachycardia, cyanosis, dyspnea, and vomiting.

Adult beriberi is categorized as dry or wet depending on cardiac involvement. Dry beriberi is characterized by symmetrical peripheral neuropathy of the distal extremities, with resulting sensory and motor impairments.

Wet beriberi includes the addition of cardiac involvement (cardiomyopathy, high-output congestive heart failure, peripheral edema, and tachycardia) .

Central nervous system involvement (ie, Wernicke-Korsakoff syndrome) occurs primarily in alcoholics.

Educational message :
Thiamine deficiency causes beriberi and Wernicke-Korsakoff syndrome.
Dry beriberi is characterized by symmetrical peripheral neuropathy; wet beriberi includes the addition of high-output congestive heart failure.

100
Q

Uworld nutrition case :

42-year-old man comes to the office due to numbness and tingling in both legs and difficulty walking for the past several months. He has also noticed that he tires more easily v,ith physical activity. His temperature is 36.8 C (98 F), blood pressure is 122/86 mm Hg, pulse is 76 /min, and respirations are 14/min. Physical examination shows conj unctiva! pallor and loss of vibration and position sensation in the bilater al lower extremities v,ith associated gait ataxia. The remainder of t he examination is within normal limits.
Which of t he following findings is most likely to be present upon further questioning of the patient?

A

Vitamin B12 (cobalamin) is obtained solely from the diet, specifically from animal products such as meat, dairy, and fish.
Vitamin B12 cannot be obtained from plant products, placing strict vegans at risk for dietary deficiency.

The body is capable of storing about 1,000 times the daily requirement; therefore, vitamin B12 deficiency develops only after the complete absence of intake for 4-5 years.

The development of anemia as severe and potentially irreversible neurologic damage can result from vitamin B12 deficiency.

Red blood cell synthesis relies on vitamin B12-dependent recycling of folate ; deficiency of either vitamin results in a megaloblastic anemia .

Neurologic sy mptoms vary in severity, but patients typically present with both motor and sensory deficits.

Impaired myelin synthesis, specifically in the dorsal and lateral columns, results in subacute combined degeneration of the spinal cord.
This condition is progressive and begins as a symmetrical neuropathy consisting of paresthesias and weakness.

Ongoing deficiency leads to loss of vibration and position sensation with the development of an ataxic gait. The longer the deficiency is untreated , the less likely it can be reversed.

Educational objective:

Vitamin B12 is obtained thro ugh the diet solely from anima l sources, which places strict vegans at risk for dietary deficiency.
This deficiency takes years to develop due to the large hepatic B12 reserve and presents with megaloblastic anemia and potentially irreversible neurologic deficits (eg, paresthesias, weakness, ataxic gait).

101
Q

The mamillary bodies

A

-are located in the brain in the hypothalamus

-responsible for memory

-in wernicke -korsakoff syndrome , they are affected (either by hemorrhage or necrosis ) —-) therefore we have memory impairment +++++

102
Q

Wernicke encephalopathy :

A

-A medical emergency

-Treated immediately to prevent Korsakoff syndrome

-Therapy :
_Infusion of thaimin over a few days
_Given along with glucose (but ALWAYS REMEMBER ,Thiamine first then glucose )

if we give glucose first —-) we will have the accumulation of pyruvate and without the enzyme that convert pyruvate into acetyl CoA —) pyruvate will transform into Lactate —–) Lactic acidosis +++++

103
Q

A 32-year-old woman in the intensive care unit is brought to the attention of the on-call physician for acute mental status changes shortly after a nurse initiated dextrose supplementation. She was admitted to the hospital 1 week ago for severe acute pancreatitis. Past medical history includes anorexia nervosa and 2 hospital admissions in the past year for alcohol withdrawal. Temperature is 37.0°C (98.6°F), pulse is 92/min, respirations are 18/min, and blood pressure is 125/85 mmHg. BMI is 17.5 kg/m2. She is confused and disoriented to time, place and person. Pupils are equal and reactive, and horizontal nystagmus is elicited on lateral gaze. Fundoscopy is normal. Motor strength is 5/5 in the upper extremities and 2/4 in bilateral lower extremities. Deep tendon reflexes of the knee and ankle are 1+ bilaterally. Rapid glucose test is 70 mg/dL. Laboratory investigations are noncontributory. Which of the following enzymes is likely inhibited considering the most likely diagnosis in this patient?

A

Pyruvate dehydrogenase

Major takeaway

Wernicke encephalopathy is a complication that results from thiamine deficiency characterized by a triad of ophthalmoplegia, ataxia and confusion. It should be suspected in patients with risk factors for malnutrition and malabsorption. It can be precipitated by glucose administration prior to thiamine administration and requires urgent thiamine supplementation, as it can progress to Korsakoff syndrome, which is irreversible.

This patient has symptoms of acute confusion and ophthalmoplegia in the setting of chronic malnutrition due to pancreatitis, chronic alcohol use, and anorexia nervosa. She most likely has Wernicke encephalopathy (WE). WE and thiamine deficiency should be considered in all patients with malnutrition and malabsorption, including patients with a history of chronic alcohol use, malabsorption syndromes, and those receiving total parenteral nutrition.

A diagnosis can be made based on clinical suspicion alone. However, confirmation with a decreased blood thiamine level, decreased erythrocyte transketolase activity, and degenerated mammillary bodies on MRI imaging can aid the diagnosis. WE is a reversible condition that requires a thiamine infusion. With a high glucose intake, the requirement for thiamine increases in order to assist in the breakdown of glucose through the Krebs cycle, and thus a glucose infusion can induce or worsen WE. If glucose (dextrose) is administered prior to thiamine replacement, it would be converted to lactic acid, and as a result, it would precipitate WE in susceptible populations such as those with chronic alcohol use and malnourished patients (e.g. anorexia nervosa).

104
Q

Where do B12 get absorbed ?

A

-In the terminal ileum ++++

105
Q

Romberg test ?

A
106
Q

A 48-year-old man comes to the clinic with generalized fatigue, numbness, and weakness in his lower limbs over the last 2 weeks. Notable past medical history includes Crohn disease, for which he underwent an ileocecal resection 6 months ago and has been receiving total parenteral nutrition (TPN) since the procedure. He additionally has a history of chronic alcohol use and has been drinking 6-7 beers daily for the past 25 years. Temperature is 37.0°C (98.6°F), pulse is 80/min, respirations are 20/min, and blood pressure is 135/85 mmHg. BMI is 19 kg/m2. He is alert, cooperative, and oriented to time, place, and person. Neurological examination reveals symmetrical peripheral neuropathy in the upper and lower extremities with defective perception of touch and vibration sensation. Motor strength is 3/5 in the lower limbs and 5/5 in the upper limbs bilaterally. Symmetrical muscle wasting is noted on both lower limbs. Deep tendon reflexes are absent at the ankles bilaterally. Romberg test is negative. Laboratory investigations are shown below:

Laboratory value Result
Complete blood count
Hemoglobin 12 g/dL
Hematocrit 40%
Leukocyte count 9,100/mm3
Platelet count 150,000/mm3
MCV 82 fL

A

Major takeaway
Dry beriberi caused by thiamine deficiency results in symmetrical distal peripheral neuropathy and motor weakness. Diagnosis of thiamine deficiency can be made by measuring serum thiamine levels or by observing an increase in erythrocyte transketolase activity after vitamin B1 administration.

Thiamine is a water-soluble vitamin that acts as a cofactor for the conversion of pyruvate to acetyl CoA. It can present as wet/dry beriberi or can cause Wernicke-Korsakoff syndrome.

Thiamine deficiency should be suspected in patients who have a history of heavy alcohol consumption and those with a prolonged history of receiving TPN. Dry beriberi is common in patients with a history of excessive alcohol intake and is characterized by distal peripheral neuropathy with a diminished perception of vibration and touch. Additionally, motor weakness and loss of deep tendon reflexes, particularly of the lower extremities, is also associated with dry beriberi. Wet beriberi includes cardiac involvement and manifests as cardiomegaly, cardiomyopathy, heart failure, peripheral edema, and tachycardia, in addition to neuropathy.

Diagnosis of dry beriberi can be made clinically in a case of high clinical suspicion (heavy alcohol intake and/or poor nutrition with concomitant peripheral neuropathy). However, confirmation of the diagnosis can be done by measuring serum thiamine levels or the activity of erythrocyte thiamine transketolase activity (ETKA). Transketolase is an enzyme involved in the HMP shunt pathway that uses thiamine as a cofactor. In thiamine deficiency, erythrocyte transketolase activity is reduced and improves after vitamin B1 administration, confirming the diagnosis.

107
Q

A 65-year-old man is brought in by his partner to the clinic when she noticed that he has been having episodes where he is disoriented and markedly irritated over the last 6 months. He has had 2-3 episodes of loose stools, sore throat, and skin rashes over his chest and arms over this same time period. He has not had a fever, headache or loss of consciousness. Nine months ago, he was diagnosed with a gastric tumor and was found to have elevated serum chromogranin levels. He declined medical intervention at the time. He does not smoke. He drinks 6-7 beers daily. Temperature is 37.0°C (98.6°F), pulse is 80/min, respirations are 20/min, and blood pressure is 125/85 mmHg. BMI is 19 kg/m2. He is oriented to place and person but is slow to answer questions. Physical examination reveals scaly erythematous patches and hyperpigmentation of his arms, chest, and skin below his knees. Pupils are equal and reactive. Extraocular movements are normal. There is no nuchal rigidity on examination. Motor strength is 5/5 in all four extremities. This patient’s symptoms will most likely respond to which of the following supplements?

A

Major takeaway
Niacin is a water-soluble vitamin formed from the precursor amino acid tryptophan. Niacin deficiency should be suspected when clinical manifestations of diarrhea, dementia and dermatitis are present in patients with a history of carcinoid syndrome, isoniazid use, chronic alcohol use, bariatric surgery, and malabsorptive symptoms.
Main explanation
This patient’s clinical features of dementia and dermatitis in the setting of a carcinoid tumor (gastric tumor with elevated serum chromogranin) suggests a niacin deficiency resulting in pellagra.

Niacin is a water-soluble vitamin that is formed from the precursor amino acid tryptophan. Tryptophan also serves as the precursor for the neurotransmitter serotonin. In patients with carcinoid syndrome, excess serotonin production leads to depletion of tryptophan, and in turn, niacin. Niacin deficiency is also likely in patients with a history of chronic alcohol use, bariatric surgery, and history of prolonged isoniazid use in the treatment of tuberculosis. Patients may present with the classic triad of symptoms, which includes dermatitis (especially in sun-exposed areas of the body), diarrhea, and dementia, known as 3D

108
Q

Dysmetria definition :

A

What is dysmetria?
Dysmetria is the inability to control the distance, speed, and range of motion necessary to perform smoothly coordinated movements. Dysmetria is a sign of cerebellar damage, and often presents along with additional signs, such as loss of balance and poor coordination of walking, speech, and eye movements. More specifically, dysmetria is a type of cerebellar ataxia, which is the general term used to describe an abnormal coordination of movements.

109
Q

A 45-year-old man goes to the clinic because of progressive gait instability, dysmetria and dysarthria for the past several months. He was a part of a local basketball team and had to quit recently because of his poor performance. Past medical history is significant for celiac disease, hypertension, diabetes, and atrial fibrillation. Current medication includes lisinopril, metformin, and warfarin. Family history is noncontributory. Vitals are within normal limits. Physical examination shows mucosal pallor and reduced muscle mass. Examination of lower extremities show bilateral motor weakness and loss of deep tendon reflexes and sensation to joint position and vibration. Current INR is 2.5. Laboratory results are shown:

Laboratory features
Laboratory value Result
Hemoglobin 9 g/dL
Leukocyte count 8,000/mm3
Platelet count 230,000/mm3
Mean corpuscular volume 85/μm3
Reticulocyte count 4%

The patient is started on high-dose tocopherol supplementation. Which of the following complications is this patient at increased risk for due to this medication?

A

Hemorrhage

Major takeaway
Vitamin E toxicity is rare but may result from high-dose supplements. It may be asymptomatic or can present with muscle weakness, fatigue, nausea, and diarrhea. In patients on warfarin, it can interfere with vitamin K metabolism and increase the risk of bleeding.
Main explanation

This patient’s neurological symptoms of gait instability, decreased position and vibration sense, and ataxia in combination with hemolytic anemia (normocytic anemia with reticulocytosis) is most likely suggestive of vitamin E deficiency secondary to long-term fat malabsorption due to underlying celiac disease.

Vitamin E is a fat-soluble vitamin that is widely available in the diet. Its deficiency is rare but can occur with fat malabsorption (e.g., cystic fibrosis and celiac disease) or abetalipoproteinemia (mutations in the microsomal triglyceride transfer protein). Its toxicity is rare but could result from high-dose supplements. Excess vitamin E may interfere with vitamin K metabolism and increase the anti-coagulation effects of warfarin, thus increasing the risk of bleeding and hemorrhage. Usually, elevated vitamin E levels have no symptoms,but they may cause muscle weakness , fatigue , nausea and diarrhea

110
Q

A 15-year-old boy is brought in by his parent to the clinic for evaluation of a skin rash over his chest and arms for the past 2 weeks. The patient has noticed that the rash becomes pruritic and painful to touch when he goes outside to play soccer. He also complains of loose stools over the last week. Three months ago, he was diagnosed with latent tuberculosis and was started on isoniazid. He and his family immigrated from India 9 months ago. Temperature is 37.0°C (98.6°F), pulse is 74/min, respirations are 16/min, and blood pressure is 115/75 mmHg. BMI is 20 kg/m2. Physical examination reveals scaly erythematous patches and hyperpigmentation of the arms, chest and skin below the knees. Motor strength is 5/5 in all four extremities. Sensation is intact throughout. Which of the following amino acids is a precursor of the vitamin that is most likely deficient in this patient?

A

Major takeaway
Clinical manifestations of pellagra include a symmetric photosensitive hyperpigmented skin rash, gastrointestinal disturbances and dementia. Prolonged isoniazid use can induce pellagra, as it interferes with niacin metabolism.

Main explanation
This patient with a photosensitive skin rash and diarrhea with a history of isoniazid use has a presentation suggestive of pellagra from niacin deficiency.

Niacin is a water-soluble vitamin that is formed from the precursor amino acid tryptophan. In patients with prolonged isoniazid use, symptoms of pellagra occur due to accelerated niacin depletion. Pyridoxine is utilized for the conversion of tryptophan to niacin. Isoniazid depletes stores of pyridoxal phosphate, accelerating tryptophan synthesis. Niacin is a component of NAD and NADP+ enzymes involved in redox reactions.

Clinical manifestations of pellagra can result from isoniazid use, bariatric surgery, anorexia nervosa or consumption of a diet deficient in niacin (corn/maize). Clinical features that are characteristic of pellagra include a symmetric hyperpigmented rash that involves sun-exposed areas of the skin particularly the neck (characteristically called a “Casal necklace”). Gastrointestinal disturbances include diarrhea and vomiting. In severe cases of prolonged deficiency, neurological symptoms include insomnia, dementia, encephalopathy, lapses in concentration and delusions.

111
Q

A 10-year-old boy is brought in by his parent to the clinic for evaluation of numbness in his feet and hands. He states that his hands and feet often have a tingling and burning sensation. He has had no abdominal pain, vomiting, constipation, or diarrhea. His past medical history includes latent pulmonary tuberculosis, for which he was started on isoniazid 3 months ago in India. His family recently immigrated from India in the past month, and the patient has continued taking his medication. His mother states he is doing well in school and is currently placed in advanced academic classes. Temperature is 37.0°C (98.6°F), pulse is 80/min, respirations are 20/min, and blood pressure is 115/65 mmHg. Neurological examination reveals symmetrical “glove and stocking” distribution peripheral neuropathy of the hands and feet, with diminished perception of touch and temperature. Motor strength is 5/5 in all four extremities, and deep tendon reflexes are 2+ in the upper and lower extremities. Which of the following could have prevented this patient’s current symptoms?

A

Major takeaway
Vitamin B6 (pyridoxine) deficiency can be due to inadequate dietary or drug-induced metabolic changes. Common drugs implicated include isoniazid, penicillamine, hydralazine and levodopa/carbidopa. Clinical manifestations of vitamin B6 deficiency are peripheral neuropathy and oral stomatitis. Complications include convulsions and sideroblastic anemia.
Main explanation
This patient with peripheral neuropathy following isoniazid use has a presentation consistent with drug-induced pyridoxine (vitamin B6) deficiency.

Pyridoxine deficiency can occur due to inadequate dietary intake but is more commonly encountered with the use of drugs that interfere with its metabolism, such as isoniazid (INH), hydralazine, penicillamine and levodopa/carbidopa. Pyridoxine is converted to its active form pyridoxal phosphate, which acts as a cofactor in transamination reactions, decarboxylation reactions, and glycogen phosphorylase functions.

The mechanism of isoniazid-induced peripheral neuropathy involves interference of INH metabolites with the metabolism of vitamin B6 (pyridoxine), resulting in decreased amounts of biologically active vitamin B6. Most patients have large enough stores of pyridoxine to prevent deficiencies; however, patients who are malnourished or have certain comorbid illnesses (e.g. diabetes mellitus) are at increased risk of deficiency. Since pyridoxine is required as a vital cofactor in the synthesis of deficiency. Since pyridoxine is required as a vital cofactor in the synthesis of various neurotransmitters, deficiencies manifest with neurologic symptoms, such as numbness, tingling, and burning sensations.

Clinical manifestations of pyridoxine deficiency include peripheral neuropathy, nonspecific glossitis, stomatitis, and hyperirritability. Complications include convulsions and sideroblastic anemia. The treatment and prevention of peripheral neuropathy caused by isoniazid is pyridoxine supplementation during treatment.

112
Q

A 25-year-old woman goes to the clinic for the evaluation of worsening acne. The lesions are painful, and the patient is concerned as the symptoms have worsened. Her acne has been present for the past 2 years. Past medical history is otherwise unremarkable. She is sexually active with a male partner and uses condoms occasionally. The patient currently uses topical erythromycin and topical tretinoin, and she washes her face twice daily with benzoyl peroxide. Vitals are within normal limits. Physical examination shows multiple large >5 mm cystic nodules scattered over the face and upper trunk. The remainder of the examination is unremarkable. The physician switches the patient to oral isotretinoin and schedules her for the next follow-up. If the patient conceives while on this new medication, which of the following would her fetus be most at risk of developing?

A

Microcephaly

Major takeaway
Isotretinoin, a derivative of vitamin A, is recommended for the management of severe acne. It is a teratogenic drug and is absolutely contraindicated in pregnancy, as it can cause both spontaneous abortions and severe life-threatening congenital malformations. Therefore, a negative pregnancy test and two forms of contraception are required before isotretinoin is prescribed.

Main explanation:
This patient’s presentation with multiple scattered nodules >5 mm is consistent with nodular acne (cystic acne). She had not responded to initial therapies, so she was started on oral isotretinoin therapy, which is naturally occurring all-trans retinoic acid, a derivative of vitamin A. It counteracts the pathogenic factors that contribute to the development of acne vulgaris and inhibits follicular keratinization, thereby loosening the keratin plugs of comedones leading to their expulsion.
In addition to its mucocutaneous side effects (e.g., cheilitis, dry skin and mucous membranes), isotretinoin is also a teratogenic drug and is absolutely contraindicated in pregnancy. Exposure to this medication during the first trimester can cause both spontaneous abortions and severe life-threatening congenital malformations, including craniofacial (microcephaly, small ears), cardiac, thymic (hypoplasia), and central nervous system (hydrocephalus) malformations. Therefore, a negative pregnancy test and two forms of contraception are required before isotretinoin is prescribed, and contraception must be started at least 1 month before isotretinoin therapy. Additionally, a monthly pregnancy test is required while on the treatment.

113
Q

Which of the following gastrointestinal secretory products are required for the absorption of vitamin B12 in the terminal ileum?

A

Intrinsic factor

Major takeaway
Gastric juice also contains a protein synthesized by parietal cells called intrinsic factor. This binds to vitamin B12 and is required for the eventual absorption of this vitamin in the terminal ileum.

Main explanation
Gastric juice contains a protein synthesized by parietal cells called intrinsic factor. This binds to vitamin B12, also known as cobalamin, and is required for the eventual absorption of this vitamin more distally in the intestine in the terminal ileum. Intrinsic factor is specialized for its role by being relatively resistant to degradation by acid and proteolytic enzymes.
Following ingestion of vitamin B12, the stomach acid releases B12 from its food carrier proteins. The free vitamin B12 binds to haptocorrin (R proteins) secreted by salivary glands, which protects B12 from acid degradation. Eventually, the pancreatic proteases degrade R proteins in the duodenum, where B12 binds to the intrinsic factor (secreted by gastric parietal cells) to protect it from pancreatic enzymes. The intrinsic factor-B12 complex is resistant to degradation from pancreatic enzymes. Finally, this complex binds to a specific receptor located in the apical membrane of epithelial cells lining the terminal ileum, where vitamin B12 is absorbed.

114
Q

A 73-year-old woman comes to a medical clinic for evaluation of easy bruising. The patient is currently undomiciled and frequently eats canned-goods for meals. The patient has a history of chronic alcohol use, intravenous drug use, and 20-pack-year smoking history. Temperature is 37.0 C (98.6°F), blood pressure is 110/70 mmHg, and pulse is 99/min. Mucous membranes are moist with evidence of swollen gums and multiple dental caries. There are scattered bilateral petechial lesions on the extremities and the trunk. There are 2 nonhealing ulcers over the sacrum that do not appear to be infected. Laboratory evaluation, including a complete blood count, is found to be within normal limits. Which of the following biochemical processes is most likely impaired in this patient?

A

response :
Hydroxylation of proline and lysine residues

Major takeaway
Scurvy (i.e. Vitamin C deficiency) results from impaired hydroxylation of proline and lysine residues during the synthesis of collagen resulting in gingivitis, mucosal hemorrhages, myalgias and subperiosteal hematomas.

Main explanation
This patient with bleeding gums, petechial hemorrhage, and poor wound healing has typical features of scurvy. In the setting of malnutrition, chronic alcohol intake and advanced age, she most likely has vitamin C deficiency.

Vitamin C is obtained primarily from dietary sources, and deficiency results from inadequate dietary intake. Although inadequate dietary intake is uncommon in most developed countries, it is seen in elderly, institutionalized patients with a history of chronic alcohol use, poor nutritional status, or those with a history of psychiatric disorders.

Vitamin C acts as a reducing agent and hydroxylates proline and lysine residues within the structure of collagen during synthesis using the enzymes prolyl and lysyl hydroxylase. Failure of this step prevents cross linking of collagen fibrils and results in the formation of reduced collagen with decreased tensile strength.

The typical presentation of vitamin C deficiency indicates microvascular bleeding due to collagen defects in blood vessels and includes gingivitis (bleeding receding gums and dental caries), perifollicular hemorrhages, cutaneous hyperkeratosis, poor wound healing and increased susceptibility to infections. Deficient collagen formation also affects osteoblast and fibroblast function, manifesting as bone pain, myalgias and subperiosteal hematomas.

115
Q

A 31-year-old man comes to the office for the evaluation of bleeding from the gums and easy bruising. The patient reports that even minor trauma causes a bruise, and he sometimes bleeds spontaneously from his gums. He also complains of frequent fatigue, generalized weakness, and poor appetite. Past medical history is significant for Crohn disease that required a partial small bowel resection a year ago. The patient has also been receiving oral antibiotic treatment for a perianal fistula for the past 3 months. Family history is noncontributory. Vitals are within normal limits. Physical examination shows multiple large ecchymoses of the lower extremities and trace bleeding from the gums. Laboratory results are shown.

Laboratory features
Laboratory value Result
Platelet count 250,000/mm3
Bleeding time 5 minutes
Prothrombin time 29 seconds
Partial thromboplastin time 43 seconds
Bilirubin
Total 0.8 mg/dL
Direct 0.2 mg/dL
Alanine aminotransferase 15 U/L
Aspartate aminotransferase 17 U/L

Which of the following is the most likely cause of this patient’s presentation?

A

Response :
Bile acid malabsorption

Vitamin K is essential for carboxylation of coagulation factors II, VII, IX and X. Its deficiency results in coagulation defects which can present with easy bruising and bleeding after mild trauma or surgery. Common predisposing factors include malabsorptive disorders and medication use (e.g. cephalosporins and warfarin). Laboratory evaluation shows a prolonged PT, mildly prolonged or normal PTT, and a normal bleeding time.

Main explanation
This patient with a history of Crohn disease and partial small bowel resection is experiencing symptoms of vitamin K deficiency most likely due to bile acid malabsorption. Bile acids are required for the absorption of fats and other nutrients, which are normally reabsorbed in the terminal ileum (frequently affected in Crohn disease), recycled in the liver and then reused in the absorptive process. Resection or inflammation of the terminal ileum results in loss of bile acids in feces, leading to fat malabsorption and deficiency of fat-soluble vitamins.
Vitamin K is a fat-soluble vitamin which is an essential cofactor for gamma-glutamyl carboxylase, an enzyme responsible for the carboxylation of coagulation factors II, VII, IX and X. Its deficiency results in coagulation defects and presents with easy bruising, hematoma formation in bones and joints after minor trauma, or prolonged bleeding after surgery. Besides Crohn disease, other causes of vitamin K deficiency include other malabsorptive conditions like cystic fibrosis, primary sclerosing cholangitis, and celiac disease. Liver failure and medications (e.g., cephalosporins, warfarin) can also contribute to the development of vitamin K deficiency.

Laboratory evaluation usually reveals a prolonged prothrombin time (PT)
mildly prolonged or normal partial thromboplastin time (PTT), and a normal bleeding time (BT), which indicates normal platelet function.

115
Q

A 28-year-old woman comes to the office because of a headache for the past 2 weeks. The patient reports that the headache is dull, intermittent, 5/10 in severity and is occasionally associated with nausea and vomiting. She has never had such a headache before. She denies fever or visual abnormalities. She states she has been eating multiple vitamins daily for the past 8 months in an effort to “become slimmer” and “build muscle.” Past medical history is significant for body dysmorphic disorder. Family history is not significant. Vitals are within normal limits. BMI is 19 kg/m2. Physical examination shows dry skin and hepatomegaly. Neurological examination is nonfocal and shows no meningeal signs. Fundoscopy reveals papilledema. MRI of the brain is unremarkable. Which of the following is the most likely cause of this patient’s condition?

A

Answer :
-VITAMIN A excess

Major takeaway
Vitamin A toxicity can occur in individuals who consume more than 10 times the recommended dietary allowance. Acute toxicity can cause blurry vision, nausea, vomiting, and vertigo, where chronic toxicity can result in intracranial hypertension, hepatomegaly and skin changes. Excess vitamin A is teratogenic during the first trimester of pregnancy.

Main explanation
This patient’s headache with signs of increased intracranial pressure (bilateral papilledema) and a normal MRI is suggestive of pseudotumor cerebri, most likely due to vitamin A toxicity. Unlike water-soluble vitamins, fat-soluble vitamins can build up in the body with excessive consumption, resulting in signs and symptoms of toxicity.

While rare, vitamin A toxicity can be seen in patients who are consuming excessive amounts of vitamins, or in those who consume foods high in vitamin A, such as animal livers. Individuals who consume more than 10 times the Recommended Dietary Allowance (RDA) of vitamin A can develop toxicity. Vitamin A toxicity can be acute or chronic, and excess vitamin A can be teratogenic.

Acute toxicity can occur with a single high dose >660,000 international units (>200,000 micrograms) and typically presents with blurry vision, nausea, vomiting, and vertigo.

Chronic toxicity (chronic ingestion of approximately 33,000 international units [10,000 micrograms] of retinol in adults) can present with hair loss, dry or peeling skin, hepatomegaly, arthralgias or pseudotumor cerebri.
Teratogenicity: Retinoic acid is known to be teratogenic in the first trimester of pregnancy and can lead to spontaneous abortions and fetal malformations (e.g., microcephaly, cardiac anomalies).

116
Q

A 56-year old woman presents to the clinic for loss of taste for the past 2 months. She states that she has been unable to taste or smell food so she has been eating much less than usual. She has also noticed that she has had hair thinning in the axillae and pubic area. Her past medical history is notable for alcoholic cirrhosis. Vitals are within normal limits. On physical exam, she has a sharply-demarcated red rash around the mouth and on the hands and buttocks. Which of the following nutrients is most likely deficient in this patient?

A

Response :
Zinc (mineral )

Major takeaway
Zinc deficiency results from malnutrition, malabsorption diseases, prolonged breastfeeding, and acrodermatitis enteropathica. It typically presents with delayed wound healing, immunosuppression, male hypogonadism, alopecia, dysgeusia, and anosmia. Since the liver plays an important role in zinc homeostasis, zinc deficiency can be seen in patients with alcoholic cirrhosis.

Main explanation
Zinc is the second-most abundant trace element in the body (after iron) and the most abundant intracellular one. Zinc is absorbed mainly in the duodenum and jejunum, and it is stored in the liver via the portal circulation. As the main organ involved in zinc metabolism, the liver plays an important role in maintaining systemic zinc homeostasis. Zinc is an essential mineral that is used by a wide variety of enzymes in the form of zinc fingers (transcription factor motifs). This role in gene transcription, therefore, makes it indispensable for growth and tissue maintenance, and its deficiency may present as growth failure, alopecia, and a decline in muscle growth. These various effects of zinc deficiency are likely caused by its suppression of IGF-1 and reduction in gene transcription. IGF-1 normally stimulates cellular proliferation and uptake of amino acids and glucose which are required by proliferating cells; therefore, in cases of zinc deficiency, rapidly proliferating cells (such as hair follicles and skin) are affected.

Zinc deficiency results from malnutrition, malabsorption diseases, prolonged breastfeeding, and acrodermatitis enteropathica (autosomal recessive inherited intestinal zinc absorption defect). Since the liver plays an important role in zinc homeostasis, zinc deficiency is commonly associated with alcoholic cirrhosis. It typically presents with delayed wound healing, immunosuppression, male hypogonadism, alopecia, dysgeusia, anosmia, impaired night vision, depressed mental function, and rash around the mouth and buttocks and on acral surfaces.

It can be diagnosed with red blood cell linoleic acid to dihomo-y-linolenic acid (LA:DLGA) ratios. The recommended treatment is zinc supplementation which can be achieved through consumption of fortified cereals, whole grains, nuts, oysters, beef,

116
Q

A 2-week-old girl is brought to the clinic by her parents for a well-infant examination. She was born at 38 weeks gestation at a tertiary care hospital and weighed 3.2-kg (7-lb 1-oz) at birth. She has been exclusively breastfeeding with good latch, voiding appropriately, and stooling daily. Her mother consumes a well-balanced diet and does not consume alcohol or smoke tobacco. Vitals are within normal limits. Physical examination is unremarkable. At this time, supplementation with which of the following is most important in this infant?

A

response : Vitamin D

Major takeaway
Breast milk is deficient in vitamin D and K. Therefore, all infants should receive an intramuscular vitamin K dose at delivery and daily vitamin D supplementation beginning within a few days after birth in exclusively breast-fed infants, to be continued until the infant is weaned.

117
Q

Iron and breast milk

A

Breast milk is low in iron content, but iron bioavailability in breast milk is high and sufficient for infants up to 4 months of age. Iron supplementation is recommended for exclusively breastfed infants after 4 months of age until solid food intake provides adequate iron.

Main explanation
Breast milk is the gold standard nutrition for an infant, as it contains adequate amounts of carbohydrates, proteins, fats, vitamins, and trace minerals. Although most of the vitamins are in sufficient amounts in breast milk, vitamin D and K are relatively insufficient even in vitamin D and K sufficient mothers. Vitamin K is supplemented at delivery via an intramuscular injection to prevent hemorrhagic disease of the newborn.
Besides exclusive breastfeeding, other risk factors for vitamin D deficiency in an infant include absence of vitamin D exposure, maternal vitamin D deficiency, infants of dark skinned mothers (low vitamin D in breastmilk) or dark skinned infants (require more sunlight exposure to produce adequate vitamin D in skin). Prolonged vitamin D deficiency can lead to the development of rickets due to inadequate mineralization of bone and cartilage. Therefore, all exclusively breastfed infants should receive 400 international units (10 micrograms) daily of vitamin D supplements, beginning within a few days after birth.
Supplementation should be continued until the infant is weaned and drinks at least 33 ounces (1 liter) of vitamin D-fortified formula or cow’s milk.

Formula fed infants do not require vitamin D supplementation because formula is already fortified with sufficient vitamin D.

118
Q

bThe first milk given by the breast milk is called :

A

-colostrum
yellowish fluid (high in immune cells and antibodies )
but low in fat
-It acts as a laxative (helps pass the first stool of the baby or meconium +++)

and within a few days after delivry , the breast starts producing milk +++ (the milk in relative to colostrum has a high in fat , contain lactose , vitamins ,micronutrients ,proteins(such as casein and maternal antibodies igA )

119
Q

Breast milk is deficient in :

A

-Vitamin D (oral supplementation daily )

-Vitamin K (systematic IM injection of vitamin K )

-Iron (supplementation after 4 months of birth in the exclusively breast fed infants )

120
Q

Nipple cracking : (Crevasses du mamelon )

A

-Nipple fissures are common, but they’re not normal. A cracked nipple is a sign that there’s too much strain on your nipple tissue. Taking steps to prevent further injury while helping the tissue heal can prevent complications associated with nipple fissures.

What causes a nipple fissure?
Nipple fissures mostly appear during pregnancy or afterward once you begin breastfeeding. Less commonly, they result from friction during exercise or a skin condition.

Pregnancy and breastfeeding (chestfeeding)
Changes in your body during pregnancy and, especially, improper nursing techniques usually cause nipple fissures. Causes include:

Engorged breasts. During pregnancy, your breasts can become overly filled with milk. The excess fluid can cause your nipple tissue to stretch and eventually crack.
Improper nursing technique. Usually, nipple fissures appear because your baby isn’t latching onto your breast to nurse correctly. Your baby may struggle to get enough milk, putting more strain on sensitive nipple tissue as they try to feed.
Breast pump misuse. You can develop nipple pain and fissures if the suction settings on your breast pump are too high or if you’re using the wrong size flanges (breast shields).
Other causes of nipple fissures
Friction during exercise. The friction created when sweaty skin comes into contact with your workout clothes can cause your nipple to crack during intense exercise. Joggers and cyclists experience nipple fissures. Surfers can get them when their exposed nipples repeatedly brush their surfboards. The effects are often worse when it’s cold, and your nipples are erect.
Skin conditions. Products like soap, lotion, and laundry detergent may contain chemicals that cause an allergic reaction when they make contact with your skin. A skin condition called eczema can also cause your skin to become dry and cracked. These skin changes can cause nipple fissures.
Diagnosis and Tests
How are nipple fissures diagnosed?
Your healthcare provider can diagnose your nipple fissures during a physical exam.

Management and Treatment
How are nipple fissures treated?
You can treat fissures at home unless you experience complications like an infection or a fissure that won’t heal.

Treating fissures caused by breastfeeding (chestfeeding)
Healing nipple fissures often involves implementing proper nursing techniques. Getting your baby to latch onto your breast with a full mouth encircling the dark tissue surrounding your nipple (areola) can ease pressure on the nipple itself. An asymmetrical latch protects the nipple. A lactation consultant can show you how.

It’s also important for a trained professional to evaluate your baby’s mouth to determine why they’re having difficulty latching properly. Often a condition called a tongue tie makes it difficult for your baby to latch properly and use their tongue correctly.

Positioning yourself comfortably so that you don’t have to change positions while you’re nursing can allow your fissures to heal, too.

If nursing is too painful, you can:

Limit the amount of time you nurse and supplement by pumping.
Alternate breasts so that you’re not putting too much strain on any one nipple.
Pump and bottle feed for a few days or longer while your nipples heal.
Consult with your provider or a lactation expert about how long you should pump and how to use a breast pump correctly to prevent nipple soreness.

In addition to ensuring that you’re breastfeeding correctly, you can:

Apply gentle over-the-counter creams and ointments. Applying small amounts of Lanolin, Purslane cream, diluted peppermint oil or Menthol essence on your nipples can soothe nipple pain from fissures and encourage healing. Wipe off any antiseptic creams or lotions before you nurse.
Rub a small amount of your milk onto your fissures. The milk from your body has antibacterial properties that can moisturize nipple fissures and help cracks heal.
Allow your nipples to air dry. Using creams and ointments while allowing your nipples to air dry can promote healing while keeping your nipples from getting overly moist. Overly moist nipples are prone to tear. Wear bras with pads made out of breathable fabric. Consider going topless when possible.
Wear breast shells in between feedings. Breast shells can prevent any painful friction between your nipples and your bra while your fissures are healing.
Massage engorged breasts to relieve pain. If your breasts are so full of milk that it’s hard for your baby to latch, consider pumping first to ease pressure and encourage lactation. You can also perform reverse pressure to soften the nipples and make it easier for your baby to latch.
Apply warm compresses or gel pads to relieve pain. Gel pads for your nipples can soothe your pain and promote healing. A homemade warm towel compress can promote healing while easing sore nipples, too.

121
Q

Is caffeine contreindicated while breastfeeding ??

A

Even high doses of caffeine can get to the breast milk and alter the baby’s physiology +++

-also smoking , alcohol intake and some medications while breastfeeding can be harmful for the baby , or even for the milk (alcohol can lower the hypothalamus secretion of prolactin and oxytocin )
maternal infections like HIV

122
Q

A 15-year-old boy is brought to the office by his parents for the evaluation of progressive gait instability, dysmetria and dysarthria for the past several years. The patient enjoys hockey but can no longer play due to his worsening symptoms. The review of systems is significant for chronic abdominal pain and fatty diarrhea. The patient is concerned that he is the shortest boy in his class. Family history is noncontributory. Physical examination shows a lean boy with mucosal pallor and reduced muscle mass. Skin examination reveals multiple tense, grouped blisters on the forearms as well as excoriations. Examination of lower extremities shows bilateral motor weakness and loss of deep tendon reflexes and loss of sensation to joint position and vibration. Laboratory results are shown below:

Laboratory features
Laboratory value Result
Hemoglobin 9 g/dL
Leukocyte count 8,000/mm3
Platelet count 230,000/mm3
Mean corpuscular volume 85/μm3
Reticulocyte count 4%

Peripheral blood smear is shown.

MRI of the brain and spine shows spinocerebellar and posterior column cells degeneration. Which of the following is the most likely cause of this patient’s condition?

A

Response :

Vitamin E deficiency

Major takeaway
Vitamin E deficiency is rare but can occur with fat malabsorption or abetalipoproteinemia. It usually manifests with hemolytic anemia, acanthocytosis, muscle weakness, and demyelination of the posterior columns and spinocerebellar tract.

Main explanation
This patient’s neurological symptoms of gait instability, decreased position and vibration sense, and ataxia in combination with hemolytic anemia (normocytic anemia with acanthocytes in peripheral smear) is most likely indicative of vitamin E deficiency secondary to long-term fat malabsorption due to underlying celiac disease (fatty diarrhea, abdominal pain and reduced growth).

Vitamin E is a fat-soluble vitamin that is widely available in the diet. Its deficiency is rare but can occur with fat malabsorption (e.g., cystic fibrosis, celiac disease) or abetalipoproteinemia (mutations in the microsomal triglyceride transfer protein). It is an antioxidant which protects the red blood cells and membranes from free radical damage. Therefore, its deficiency usually presents with hemolytic anemia, acanthocytosis and neurological manifestations including ataxia (due to spinocerebellar tract degeneration), loss of position and vibration sense (due to dorsal column degeneration) and loss of deep tendon reflexes (due to peripheral nerve degeneration). Other neurologic manifestations may include ophthalmoplegia, pigmented retinopathy, night blindness,dysarthria, dystonia, and tremor. Diagnosis is made by measuring ɑ-tocopherol levels (ɑ-tocopherol is another name for vitamin E) in the serum.

Vitamin E deficiency should be differentiated from vitamin B12 deficiency, which presents with similar neurological symptoms but would instead show megaloblastic anemia and hypersegmented neutrophils on peripheral smear and elevated serum methylmalonic acid levels.

123
Q

Stomatitis :

A

What is stomatitis?
Stomatitis refers to inflammation and redness of the oral mucosa that can lead to pain and difficulty talking, eating, and sleeping. Stomatitis can affect the inner cheeks, gums, inner lips, and tongue. The inflammation causes the formation of single or multiple painful mouth ulcers as well as white lesions. The two main types of stomatitis include herpes stomatitis (i.e., cold sore) and aphthous stomatitis (i.e., canker sore).

Is stomatitis a sexually transmitted disease (STD)?
Stomatitis is not a sexually transmitted disease (STD). In some cases, stomatitis can be caused by the herpes simplex virus, which can be transmitted sexually. However, herpes simplex virus can also be transmitted through close contact like sharing eating utensils or kissing.

What causes stomatitis?
The causes of stomatitis depend on the specific subtype. Herpes stomatitis is commonly caused by the herpes simplex virus (HSV-1), whereas aphthous stomatitis is typically caused by poor oral hygiene or trauma to mucous membranes. Other causes of stomatitis include local infection (e.g., Candida albicans, bacteria); systemic diseases (e.g., Behçet disease, inflammatory bowel disease); nutritional deficiencies (e.g., vitamin B12 deficiency); physical or chemical irritants (e.g., oral care products), or an allergic reaction. Additionally, tobacco use, irritating foods, chemotherapy and radiation therapy can all increase the risk of developing stomatitis. Nonetheless, it is important to note that many causes of stomatitis are idiopathic or unknown.

What are the signs and symptoms of stomatitis?
The signs and symptoms of stomatitis include red patches, blisters, and swelling of the mouth. These symptoms may be accompanied by oral dysaesthesia, which is a burning sensation in the mouth, and recurring ulcers. The accompanying ulcers usually have a white or yellow layer with a red base and are typically inside the lips, cheek, gum, or on the tongue. The ulcerative lesions can also cause pain with eating, drinking, or swallowing, which may potentially lead to dehydration. Other symptoms of stomatitis include drooling, pain, and swelling of the gums.

How is stomatitis diagnosed?
Stomatitis can be diagnosed through a thorough physical examination by a clinician. They may take note of the severity of pain and presence of triggers, such as specific foods, toothpaste, or drugs. The clinician may assess for potential underlying causes, such as tobacco use, presence of sexually transmitted infections, or immune system compromise that may predispose the person to infection. If the symptoms of stomatitis are recurrent, aphthous stomatitis and herpes stomatitis are typically the cause. Having a history of diabetes or HIV infection, or using antibiotics recently should increase suspicion of a Candida infection as cause of stomatitis. If those with acute stomatitis experience no signs, symptoms, or risk factors for systemic illness, they typically require no testing. If symptoms of stomatitis are prolonged and severe, testing can be performed to confirm the cause. Subsequent testing includes bacterial and viral cultures, laboratory tests (e.g., complete blood count, serum iron, ferritin, vitamin B12, folate, zinc, and tissue transglutaminase antibody for celiac disease) are conducted. If there is no obvious etiology and the lesions are persistent, a biopsy can be conducted at the periphery of normal and abnormal tissue.

How is stomatitis treated?
Treatment for stomatitis depends on the underlying cause. For cases of herpes stomatitis, antiviral medications (e.g., acyclovir, valacyclovir) may be administered. Additionally, over-the-counter ointment (e.g., docosanol) can assist in shortening the length of infection. Over-the-counter medications, such as acetaminophen, can be used to reduce the pain associated with stomatitis. Aphthous stomatitis is rarely severe and treatment may not be necessary, especially if it clears on its own within 1-2 weeks.

For large and persistent sores, mouth rinses containing chlorhexidine, an antimicrobial; dexamethasone, which can reduce inflammation; or lidocaine for pain relief, can be prescribed by a clinician. Topical creams containing benzocaine, fluocinonide, and hydrogen peroxide can also be used to relieve pain and accelerate healing. Lastly, for uncommonly severe cases of aphthous stomatitis, chemical cauterization (i.e., burning or destruction of tissue) may be indicated. Debacterol and silver nitrate are two chemicals used for cautery of sores in the mouth, and can reduce healing time. In all cases, since dehydration is a risk, the individual should be encouraged to drink more fluids, specifically water. Individuals should also be encouraged to reduce the intake of acidic foods and beverages to reduce further irritation.
What are the most important facts to know about stomatitis?
Stomatitis refers to inflammation of the oral mucosa, which presents with ulcers that can cause pain and difficulty drinking and eating. Ulcer(s) can be present on the inner lips and cheeks, on the gums, or on the tongue and are caused by infection, irritants, trauma, or allergic reactions. A common cause of herpes stomatitis, a subtype of stomatitis, is herpes simplex virus. Another common type of stomatitis is apthous stomatitis, or a canker sore, which is caused by poor oral hygiene or trauma to the mucous membranes. Diagnosis involves history taking, physical exam, and if necessary, laboratory testing to determine the cause. Treatments for stomatitis include oral rinses, topical ointments, antivirals if the etiology is viral, and anti-inflammatory medication to reduce the swelling and pain.

124
Q

A 42-year-old woman comes to the clinic with generalized fatigue and a sore throat for the past week. She has a 15-year history of celiac disease that is poorly controlled with diet, and she frequently has large, bulky stools multiple times per day. Past medical history also includes seborrheic dermatitis. Temperature is 37.0°C (98.6°F), pulse is 96/min, respirations are 20/min, and blood pressure is 135/85 mmHg. BMI is 19 kg/m2. She is alert, cooperative, and oriented to time, place, and person. Oral examination reveals hyperemia of pharyngeal membranes, edema of mucous membranes, glossitis and stomatitis.

On cardiac auscultation, S1 and S2 are normal, and no murmurs or gallops are heard. Laboratory investigations are shown below:

Laboratory value Result
Complete blood count
Hemoglobin 9 g/dL
Leukocyte count 5,000 /mm3
Platelet count 150,000/mm3
MCV 88 fL

An underlying water-soluble vitamin deficiency is suspected. Which of the following biochemical conversions are impaired, considering the most likely deficiency in this patient?

A

Answer :
Succinate to fumarate

Major takeaway
Riboflavin deficiency is a water-soluble vitamin which is a component of FAD and FMN, coenzymes involved in the TCA cycle and the electron transport chain. Clinical manifestations of riboflavin deficiency include stomatitis, glossitis, cheilitis, seborrheic dermatitis, and normocytic anemia.
Main explanation
This patient likely has a riboflavin deficiency, considering her history of active celiac disease and seborrheic dermatitis, with physical examination findings of stomatitis, glossitis and cheilitis.
Riboflavin is a water-soluble vitamin that is the precursor of coenzymes FMN (flavin mononucleotide) and FAD (flavin adenine dinucleotide). FAD is a known component of the succinate dehydrogenase complex that is involved in the conversion of succinate to fumarate, a step in the TCA cycle and a component of the electron transport chain.

Patients with malabsorptive syndromes like active celiac disease, short bowel syndrome and anorexia nervosa are at an increased risk of developing riboflavin deficiency. Lactose intolerance and a diet deficient in dairy increases the risk of riboflavin deficiency, as dairy products are a rich source of vitamin B2. Clinical manifestations of riboflavin deficiency include stomatitis, cheilitis and hyperemia of pharyngeal mucous membranes. Laboratory findings typically reveal normocytic anemia

125
Q

Hemochromatosis :

A

-A metabolic disorder where the body absorbs too much iron fron the food we eat

-too much iron get accumulated in the skin , liver , heart , joints ,pancreas (the process of deposit of iron in the organs is called hemosiderosis +++)

-HEMO : elevated iron in the blood (that will poison the other tissues )

-CHROMAT : color or darkening of the skin (happens when there is deposit of iron on it )

-OSIS : disorder

-physiology of iron :
normally we lose a quantity or iron each day (1mg/day)either by sweat , skattering of skin cells or in the GI tract ,
through the diet , we take 10-20 mg/ day of iron and absorb 1-2 mg / day

people with hemochromatosis absorb too much iron

-Iron can give free radicals by fenton reaction (so any excess can give free radicals ) —-) cellular damage by free radicals deposition —-) cell death —-) tissue fibrosis +++

126
Q

primary and secondary Hemochromatosis :

A

-Primary hemochromatosis : hereditary (a gene mutation ) that don’t regulate the amount of iron absorbed so more iron is absorbed through the gut and deposit

-Secondary hemochromatosis : example frequent blood transfusion

an important note to know about men and women in hemochromatosis : - men usually manifests hemochromatosis at age 50 , but women manifests it later than men because of menstruation in women (they get rid of a little bit of iron )

127
Q

case osmosis :
A 45-year-old man comes to his primary care provider for evaluation of fatigue, which began six months ago. He has felt more tired than usual and has had decreased libido over the same time period. The patient attributes these symptoms to recent stress at work as an attorney. Past medical history is noncontributory and he does not drink alcohol or smoke cigarettes. Temperature is 36.5°C (97.7°F), pulse is 60/min, respirations are 14/min, and blood pressure is 130/82 mm Hg. Physical examination shows diffuse hyperpigmentation of the skin. The liver is palpated 4 cm below the right costal margin. Mild testicular atrophy is noted. Laboratory testing reveals the following results:

If untreated, this patient’s condition would most likely give rise to which of the following liver findings?

A

Answer : Liver nodule and elevated alpha-fetoprotein levels

Major takeaway
Hemochromatosis is an autosomal recessive condition characterized by iron overaccumulation within the body. Patients with hemochromatosis are at increased risk of developing hepatocellular carcinoma, which can manifest as a liver nodule and elevated alpha-fetoprotein levels.

Main explanation
This patient is presenting with fatigue, hepatomegaly, hypothyroidism, skin hyperpigmentation, and elevated glucose levels. He likely has underlying hemochromatosis. If untreated, the condition can give rise to hepatocellular carcinoma, which manifests as a liver nodule and elevated alpha-fetoprotein levels.

Hemochromatosis is an autosomal recessive disease caused by mutations in the HFE gene that lead to increased iron absorption in the small intestine. Hemochromatosis is mostly asymptomatic early in life, but as toxic levels of iron accumulate in the body, it can become symptomatic, usually between the 3rd and 5th decade of life.

he various organs that iron deposits in correlate with the clinical features of the disease: pancreatic accumulation leads to diabetes mellitus, dermal deposition results in hyperpigmented and bronze skin, pituitary damage causes decreased libido and testicular atrophy, joint deposition leads to arthralgias, thyroid deposition leads to hypothyroidism, and hepatic deposition results in hepatomegaly with eventual cirrhosis. The risk of hepatocellular carcinoma (HCC) in patients with hemochromatosis is up to 200 times higher than the general population. In fact, HCC causes nearly 50% of deaths in patients with hemochromatosis. HCC typically presents as weight loss, cachexia, a liver nodule, and elevated alpha-fetoprotein levels.

128
Q

The coloration used in the liver biopsy to detect iron is :

A

-Prussian blue staining of hepatocytes +++

129
Q

Metformin and B12 metabolism ???

A

metformin interferes with the metabolism of B12

130
Q

An 85-year-old man is brought in by his niece to the clinic after finding him confused outside his house. He denies the incident and says that he was “just walking his dog around the neighborhood” after having a meal with his friends. After the consultation, his niece states that “He doesn’t own a dog, he lives alone, and most of his meals consist of reheating canned foods.” Past medical history includes hypertension, hypercholesterolemia and type 2 diabetes. Medications include hydrochlorothiazide, atorvastatin and metformin. Family history is noncontributory. He has been drinking 5-6 beers daily for the past 30 years. Temperature is 37.0°C (98.6°F), pulse is 92/min, respirations are 17/min, and blood pressure is 135/85 mmHg. He is oriented to place and person but not time. Immediate and delayed memory recall is poor. Pupils are equal and reactive, and horizontal nystagmus is elicited on lateral gaze. Fundoscopy is normal. There is no nuchal rigidity on examination. Motor strength is 5/5 in the upper extremities and 2/4 in bilateral lower extremities. Deep tendon reflexes of the knee and ankle are 1+ bilaterally. MRI reveals necrosis in the mamillary bodies and medial thalamus. Which of the following is the most likely diagnosis?

A

Answer :
Korsakoff syndrome
Major takeaway
Patients with Korsakoff syndrome can present with retrograde amnesia, anterograde amnesia and confabulations. Although most symptoms of thiamine deficiency, such as ataxia, peripheral neuropathy and ophthalmoplegia, are reversible with thiamine administration, Korsakoff syndrome is an irreversible disease.

Main explanation
This patient’s confusion and ophthalmoplegia, in the setting of chronic alcohol use, malnutrition, and MRI findings of degeneration of mammillary bodies, are consistent with a diagnosis of Korsakoff syndrome.

Korsakoff syndrome is a complication that results from thiamine (vitamin B1) deficiency. Thiamine deficiency should be suspected in patients with a history of chronic alcohol use, IBD, malnutrition, malabsorption syndromes, and those on prolonged total parenteral nutrition. Thiamine is a cofactor involved in the function of enzymes such as pyruvate dehydrogenase (links glycolysis to TCA cycle) and alpha ketoglutarate dehydrogenase (TCA cycle). These enzymes play a crucial role in glucose metabolism. In the absence of thiamine, glucose metabolism is impaired, which can lead to ATP depletion and result in neuronal dysfunction/death.

Korsakoff syndrome is characterized by anterograde (inability to form new memories) and retrograde amnesia. Confabulation is also commonly encountered, wherein patients create false stories to make up for the lapses in memory. A clinical diagnosis is often sufficient to make a diagnosis. Radiological investigations, such as an MRI, can help differentiate causes of dementia that can mimic the presentation of Korsakoff syndrome. MRI scan will reveal necrosis or hemorrhage of the mammillary bodies and other periventricular regions including the anterior and dorsomedial thalamic nuclei.

Laboratory investigations will reveal a reduced blood thiamine concentration. The memory deficit in Korsakoff syndrome is irreversible and generally permanent. Thiamine should be administered to all patients who present with Wernicke encephalopathy to prevent the irreversible neuronal damage that could result in Korsakoff syndrome.

131
Q

A 45-year-old man comes to the clinic in winter for evaluation of recurrent pruritus, flushing and paraesthesias over his face and chest. He was diagnosed with hypertension and hypercholesterolemia last week, and he was started on hydrochlorothiazide and supplemental niacin. He states the symptoms typically occur 30 minutes after taking his medications at night. He does not have trouble breathing during these episodes. His past medical history includes seasonal rhinitis that is well managed with cetirizine. Temperature is 37.0°C (98.6°F), pulse is 82/min, respirations are 20/min, and blood pressure is 135/95 mmHg. BMI is 32 kg/m2. Physical examination is within normal limits. Which of the following is the most likely mechanism for this patient’s adverse reaction?

A

Prostaglandin-mediated vasodilation

Major takeaway
Niacin can be used as a lipid-lowering agent and is effective in reducing triglyceride and LDL levels. Some patients may experience facial flushing, paresthesias and pruritus following treatment with niacin; pretreatment with aspirin or NSAIDs can prevent these prostaglandin-mediated adverse effects.

Main explanation
This patient with symptoms of flushing, paraesthesia and pruritus after initiation of a lipid-lowering agent (niacin) has a presentation consistent with niacin toxicity.

Niacin is a lipid-lowering agent that inhibits lipoprotein lipase in adipose tissue and reduces hepatic VLDL synthesis. However, its use is limited to those with intolerance to standard therapy with statins due to its poor side effect profile. Clinical features associated with niacin toxicity include flushing (erythema, itching, warmth) of the face, neck and chest, paresthesias and pruritus. Other symptoms include nausea, vomiting and diarrhea. Flushing is mediated by prostaglandin (PGD2) release resulting in vasodilation of skin arterioles, which commonly lasts up to 30 minutes after niacin administration. While the flushing can be concerning to the patient, it is self-limiting and does not require medication; however, it can typically be prevented by pretreatment with aspirin or other NSAIDs due to their anti-prostaglandin properties.

Niacin can also lead to hyperglycemia, glucose intolerance and insulin resistance, and therefore, its use is avoided in susceptible patients and those with overt diabetes mellitus. It can cause hyperuricemia and can precipitate acute gout flares in patients with a history of gouty arthritis. With prolonged use, niacin is also known to cause hepatotoxicity.

132
Q

A 25-year-old woman comes to the clinic with generalized fatigue over the last 2 months. She has noticed an inability to concentrate and has recently noticed she becomes short of breath after climbing 3 sets of stairs to her apartment. She denies being depressed or having suicidal thoughts. Five months ago, she was initiated on phenytoin after 2 episodes of generalized tonic-clonic seizures. She recently initiated a high-protein diet consisting mainly of meat and animal products with minimal vegetables. Temperature is 37.0°C (98.6°F), pulse is 96/min, respirations are 20/min, and blood pressure is 125/85 mmHg. BMI is 23 kg/m2. Physical examination is within normal limits. Thyroid function tests are normal. Laboratory testing reveals a hemoglobin of 9.3 g/dL. A peripheral blood smear is performed and shows the following:

Which of the following enzymes is inhibited due to the vitamin deficiency most likely responsible for this patient’s symptoms?

A

Major takeaway
Folic acid is converted to tetrahydrofolate, which acts as a coenzyme for methylation reactions (methionine synthase) and in the synthesis of nitrogenous bases in DNA and RNA. Risk factors associated with folate deficiency include use of medications that inhibit folate synthesis (methotrexate, phenytoin, and sulfonamide). Patients with folate deficiency may present with signs of anemia, and peripheral blood smear would reveal hypersegmented neutrophils, suggestive of megaloblastic anemia.

Main explanation
This patient with a peripheral smear revealing a hypersegmented neutrophil (suggesting macrocytic, megaloblastic anemia) in the setting of phenytoin use has a presentation suggestive of folate deficiency.

Folic acid deficiency can be a result of inadequate dietary intake (major dietary sources include green leafy vegetables), use of medications that inhibit folate synthesis (methotrexate, sulfonamides and phenytoin), and reduced absorption (malabsorptive syndromes like Crohn disease). Excessive alcohol consumption also inhibits folate absorption. This patient who was recently started on phenytoin is likely experiencing folate deficiency because of its impairment of folic acid absorption in the jejunum. Folate is converted to tetrahydrofolate, which acts as a coenzyme for methylation reactions (methionine synthase) and the synthesis of nitrogenous bases (e.g. thymidine) in DNA and RNA.

Clinical manifestations of folate deficiency include oral involvement (stomatitis and glossitis) and hematological manifestations (macrocytic, megaloblastic anemia). Homocystinuria that develops in the absence of folate can result in a hypercoagulable state predisposing patients to thrombotic events (e.g., MI, stroke).

133
Q

A 3-year-old boy in a refugee camp is brought to a volunteer clinic in sub-Saharan Africa with failure to thrive. His mother states he has been small since birth and has never seen a doctor. Height and weight are at the 5th percentile. On physical exam, he is very lethargic and severely emaciated with redundant skin folds. He is noted to have little subcutaneous fat. He is admitted for supportive treatment. Which of the following best describes the pathophysiology of this patient’s condition?

A

Global nutrient deficiency

Major takeaway
Marasmus is severe malnutrition disorder that results from overall inadequate calorie and nutrient intake. Unlike Kwashiorkor, Marasmus does not cause edema. It presents with an emaciated appearance, disproportionately large head, severe muscle wasting, subcutaneous fat loss, irritability, weakness, and lethargy.

Main explanation
This patient in a refugee camp that is likely resource-deprived is suffering from Marasmus. Marasmus is severe malnutrition disorder that results from overall inadequate calorie and nutrient intake. As this results in a chronic negative energy balance, the body compensates with decreased physical activity (lethargy), decreased basal metabolic rate, growth retardation, and weight loss. It presents with an emaciated appearance, disproportionately large head, severe muscle wasting, subcutaneous fat loss, irritability, weakness, and lethargy. Unlike Kwashiorkor, which is characterized by adequate caloric intake with severely deficient protein intake, Marasmus does not present with edema

Treatment includes prophylactic antibiotics, electrolyte correction, hydration, and gradual protein refeeding. Excessive rapid refeeding can lead to protein catabolism, which can cause urea accumulation that may overwhelm the liver and ultimately result in liver failure.

134
Q

A group of investigators are studying the absorption sites of various vitamins and minerals in the digestive tract. Which of the following is the site of absorption of vitamin B12?

A

Answer:
Terminal ileum

Major takeaway
Vitamin B12 is absorbed in the terminal ileum, iron is absorbed in the duodenum, and folate is absorbed in the duodenum and jejunum.

Main explanation
Vitamin B12 (cobalamin) is absorbed in the terminal ileum along with bile salts, which requires an intrinsic factor. Iron is absorbed as ferrous Fe2+ in the duodenum and folate is also absorbed in the small bowel, primarily in the duodenum and jejunum. It is important to note that parietal cells located in the stomach secrete the intrinsic factor, which is a protein that binds to vitamin B12. B12 must be attached to the intrinsic factor for it to be efficiently absorbed, as receptors on the enterocytes in the terminal ileum of the small bowel only recognize the B12-IF complex. Furthermore, the intrinsic factor protects vitamin B12 from catabolism by intestinal bacteria.

135
Q

A 32-year-old woman comes to her primary care physician with worsening depression and frequent falls. She has had difficulty sleeping at night and has frequently been stumbling over herself while walking. The patient states “my legs feel numb all the time.” Her partner, who is also present during the visit, states that the patient has not been herself lately and has been eating less since her mother passed away. Past medical history is notable for Crohn disease, depression, and Graves disease. She drinks alcohol socially, occasionally smokes marijuana, and adheres to a vegan diet. Her temperature is 37.0°C (98.6°F), pulse is 67/min, respirations are 14/min, blood pressure is 110/74 mmHg, and O2 saturation is 98% on room air. Physical examination is notable for a sad affect, an ataxic gait, as well as paresthesias and hyperactive deep tendon reflexes in the lower extremities bilaterally. Laboratory testing reveals the following results:

Laboratory Value Result
Hemoglobin 10.5 g/dL
Leukocyte count 1,000/mm3
Platelet count 90,000/mm3
Mean corpuscular volume 115 μm3

Which of the following is the most likely etiology of this patient’s symptoms?

A

answer : Cobalamin deficiency

Major takeaway
Vitamin B12 deficiency causes subacute combined degeneration of the spinal cord, resulting in findings of gait ataxia, paresthesias, and impaired position/vibration sense.
Main explanation
This patient with a history of Crohn disease presents with findings of depression, ataxia, hyperreflexia, and paresthesias. She also consumes a vegan diet and is found to have a macrocytic anemia, thrombocytopenia, and leukopenia on her complete blood count. Together these findings are consistent with vitamin B12 deficiency and subsequent subacute combined degeneration of the spinal cord.
Vitamin B12, or cobalamin, is a water-soluble vitamin essential for proper neuronal functioning. It may become deficient due to malabsorptive disease (e.g., Crohn disease, pernicious anemia), dietary restriction (e.g. veganism), or medications (e.g., metformin, proton pump inhibitors) that interfere with cobalamin absorption. When physiologic stores of vitamin B12 are low, levels of methylmalonic acid increase. This accumulation causes degeneration of the myelin sheath of the spinocerebellar, lateral corticospinal, and dorsal column pathways, resulting in subacute combined degeneration (SCD). SCD manifests with symptoms of gait ataxia, paresthesias, and impaired position/vibration sense. Furthermore, SCD can cause neuropsychiatric symptoms including worsening depression or signs of dementia.

Additional signs of vitamin B12 deficiency include megaloblastic anemia, thrombocytopenia, and leukopenia, since vitamin B12 is essential for nucleotide synthesis and cell division.

136
Q

A 45-year-old man is brought to the emergency department by the local paramedics after he was found unconscious on the road side. The patient is covered in vomit, urine, and stool. The patient lives in a homeless shelter and has had several emergency department visits for alcohol intoxication. Past medical history is significant for chronic hepatitis C infection. Temperature is 37.2°C (99.0°F), pulse is 130/min, respirations are 19/min, and blood pressure is 114/64 mmHg. Physical examination shows hepatomegaly and dry scaly skin. Eye examination findings are shown below:
This patient’s eye examination findings are most likely related to the deficiency of which of the following?

A

Major takeaway
Vitamin A deficiency can result from malabsorptive conditions, poor nutrition and liver disorders. It usually manifests as xerophthalmia, night blindness, dry, scaly skin and immunosuppression.
Main explanation
The patient is presenting with bitot spots and dry scaly skin on a physical examination, which is suggestive of vitamin A deficiency, most likely due to poor nutrition/low socioeconomic status and liver disorder in this case. Moreover, excess alcohol consumption can deplete vitamin A stores.
Apart from poor nutrition (low socioeconomic status) and in liver disorders (e.g. cirrhosis), vitamin A deficiency is common in patients with malabsorptive conditions (e.g. celiac disease and giardiasis). Its deficiency can lead to the development of xerophthalmia, which presents with bitot spots (areas of abnormal squamous cell proliferation and keratinization of the conjunctiva), progressive corneal xerosis (dryness) and keratomalacia (corneal degradation). Vitamin A is a constituent of visual pigments, and its deficiency can also lead to night blindness and retinopathy.

Dermatological manifestations include hyperkeratosis, follicular hyperkeratosis, and the destruction of hair follicles. Vitamin A is also vital for the humoral and cell-mediated immune system, and its deficiency can lead to immunosuppression and increase the susceptibility to developing infections.

137
Q

DMARDs : disease modifying anti-rheumatic drugs

A

-Biologic DMARDs :developed from micro-organisms , animals , or humans

-Non-biologic DMARDs :chemically synthesized in the laboratory

138
Q

There are 4 bases of DNA : -Purines and Pyrimidine

A

-Purine : A , G

-Pyrimidine : C,T (require folate B9 for their synthesis +++)

139
Q

Rheumatoid arthritis (PR ) :

A

-chronic , progressive , inflammatory disorder

-impairs synovial joints and sometimes skin and lungs

-we can slow the process of inflammation in the joints by using non biologic DMARDS( example :Methotrexate ////)

140
Q

Methotrexate is teratogenic :

A

-if used during pregnancy , it can lead to neural tube defects ++++ ( spina bifida or anencephaly )

CONTREINDICATED In pregnancy +++++

-

141
Q

A 25-year-old primigravida at 40 weeks gestation comes to the emergency room in active labor. She received inconsistent prenatal care and did not take daily prenatal vitamins. Her past medical history includes rheumatoid arthritis managed with methotrexate, which she used throughout pregnancy. She did not smoke, consume alcohol or use illicit drugs during the course of her pregnancy. Temperature is 37.0°C (98.6°F), pulse is 92/min, respirations are 20/min, and blood pressure is 125/85 mmHg. She delivers a male infant with APGAR scores of 8 and 9 at 1 and 5 minutes respectively. Weight is 2.5 kg. Considering her past medical history, which of the following embryological defects is likely in this newborn?

A

Abnormal neural tube closure

Major takeaway
Folate supplementation in pregnancy is recommended as a component of preconception counselling for women who are capable of conception or are attempting conception.There exists an increased risk of folate deficiency in pregnancy due to increased requirements. When coupled with poor dietary intake or use of offending medications, neural tube defects including anencephaly and spina bifida can occur.

Main explanation
This pregnant patient has a history of rheumatoid arthritis managed with methotrexate, and she received minimal prenatal care, including lack of prenatal vitamin use. Methotrexate is contraindicated in pregnancy as it inhibits dihydrofolate reductase, arresting DNA synthesis. This history puts the neonate at an increased risk of developing neural tube defects due to folate deficiency. Folic acid is a water-soluble vitamin that is converted to tetrahydrofolate (THF), which acts as a methylation donor for reactions that involve the synthesis of nitrogenous bases for DNA and RNA.
Pregnancy is a period of increased need for folate supplementation, as the requirements of folic acid increases. Supplemental folate administration is given to all women who are attempting conception or are capable of conception as a component of preconception counselling because the neural tube begins development as early as the third week of gestation. Offending medications that can impair folic acid synthesis, function, or absorption need to be stopped and switched to alternatives for the duration of pregnancy. In the absence of folate supplementation and use of offending medications that inhibit folic acid absorption or function (e.g.,methotrexate, phenytoin, and carbamazepine), neurulation is impaired.

Neural tube defects are congenital anomalies that develop when the neural tube fails to close normally during the third and fourth weeks of gestation. Neural tube defects may be open (e.g., myelomeningocele, meningocele, encephalocele, and anencephaly) or closed (e.g., spina bifida occulta). Spina bifida is a group of neural tube defects that result from failure of posterior neural tube closure secondary to folate deficiency. Prenatal evaluation of all neural tube defects would reveal an increase in fetal alpha fetoprotein.

142
Q
A
143
Q

What vitamin does the macrophages in sarcoidosis produce ?

the granulomas of sarcoidosis are non casating (no necrosis in the center ) , the granulomas have T cells in the periphery and macrophages in the center

A

-They produce vitamin D and therefore elevates the level of calcium in the blood +++

-patients with sarcoidosis may also have elevated ACE

144
Q

A 29-year-old woman is brought to the emergency department by her partner for evaluation of progressive confusion. She also complains of nausea and vomiting for the past 24-hours. Past medical history is significant for asthma and a chronic cough for the past year. The patient has had 2 episodes of uveitis in the past 6 months treated with topical therapy. Current medications include vitamin supplementation and inhaled albuterol as needed. Family history is noncontributory. She does not use tobacco, alcohol, or illicit drugs. Temperature is 37.2 C (98.9 F), pulse is 98/min, respirations are 18/min and blood pressure is 100/60 mmHg. Physical examination shows dry mucous membranes and multiple tender pink-to-reddish nodules below the knee. Chest x-ray shows bilateral hilar lymphadenopathy. Laboratory studies are performed and shown below. Serum ACE levels are increased. HIV testing is negative. Which of the following serum substance levels is most likely to be increased in this patient?

Laboratory value Result
Serum chemistry
Sodium 132 mEq/L
Potassium 4.2 mEq/L
Chloride 95 mEq/L
Calcium 11.8 mg/dL
Creatinine 0.8 mg/dL
Blood urea nitrogen 20 mg/dL

A

Response : 1,25-hydroxyvitamin D

Major takeaway
Hypercalcemia from excessively activated vitamin D (i.e. 1,25 hydroxyvitamin D) can occur in patients with chronic granulomatous disease, such as sarcoidosis and lymphoma, wherein 1,25-hydroxyvitamin D is synthesized in the granuloma or lymphoma, respectively.

(macrophages in the granulomas of sarcoidosis secrete an enzyme 1 alpha hydroxylase that activates vitamin D +++++
therefore active vitamin D increase the absorption of calcium and phosphore in the GIT

145
Q

A 68-year-old woman comes to the clinic with generalized fatigue over the last 2 months. She has also noticed painful mouth ulcers. She states she has been consuming a “tea and toast” diet for the past 6 months in an effort to be “healthier.” Her other medical conditions include type 2 diabetes mellitus, hypertension, hypercholesterolemia and psoriasis. Her current medications include lisinopril, methotrexate, hydrochlorothiazide, NPH insulin and atorvastatin. Temperature is 37.0°C (98.6°F), pulse is 94/min, respirations are 18/min, and blood pressure is 135/95 mmHg. She is oriented to time, place and person. The patient has conjunctival pallor. Neurologic examination shows normal motor strength but decreased touch and pain sensation in the bilateral lower extremities. Complete blood count reveals a hemoglobin of 9 g/dL. HbA1c level is 9.4%. Thyroid function tests are within normal limits. Serum homocysteine levels are elevated, and serum methylmalonic acid levels are within normal limits. Peripheral smear reveals hypersegmented neutrophils and macrocytosis. A deficiency of which of the following vitamins are responsible for this patient’s symptoms?

A

answer : folate deficiency

Major takeaway
Macrocytic, megaloblastic anemia has a broad differential that should include folate and vitamin B12 deficiency. Laboratory investigations that differentiate between the deficiencies of these water-soluble vitamins include serum homocysteine levels and serum MMA levels. Whereas MMA levels would be normal in folate deficiency, MMA levels are typically elevated in vitamin B12 deficiency.

Main explanation
This patient has a history of methotrexate use, restricted diet, symptoms of anemia (e.g., fatigue, tachycardia, pallor), and laboratory investigations indicating raised homocysteine levels and normal methylmalonic acid levels. This presentation is suggestive of a folate deficiency.
The timeline of symptom onset is important when distinguishing between folate deficiency and vitamin B12 deficiency, since both cases result in a macrocytic anemia. Most vitamin B12 reserves are stored in the liver, and unless dietary intake or absorption ceases, vitamin B12 deficiency will not develop for one to two years or longer. In contrast, folate stores can be depleted over weeks to months, especially in conditions that necessitate a higher folate requirement, such as pregnancy, or in patients taking antifolate medications (e.g., methotrexate, anticonvulsants).44
Macrocytic, megaloblastic anemia has a broad differential which includes folic acid and vitamin deficiencies. Folate and vitamin B12 play key roles in the synthesis of nitrogenous bases in DNA and RNA. A deficiency in either of these vitamins can impair hematopoietic cell synthesis and result in macrocytosis. Vitamin B12 deficiency is also more likely to be associated with neurological deficits including dementia, peripheral neuropathy and subacute combined degeneration, as it serves as a cofactor for methylmalonyl-CoA mutase, which converts methylmalonyl-CoA to succinyl-CoA. In cobalamin’s absence, methylmalonic acid (MMA) accumulates and disrupts myelin synthesis. In this patient’s case, however, peripheral neuropathy is more likely to be a result of diabetic neuropathy, as suggested by her elevated HbA1c level, as opposed to a vitamin deficiency.

Laboratory investigations help differentiate vitamin B12 deficiency from folate deficiency. Serum methylmalonic acid levels and homocysteine levels would be elevated in vitamin B12 deficiency, whereas in folate deficiency although homocysteine levels are elevated, MMA levels would be normal.

146
Q

Substantial evidence supports the advantages of breastfeeding over breast milk substitutes (e.g. formula) or early addition of foods. Which of the following statements is most appropriate regarding the recommendations and benefits of breastfeeding?

A

Response :
The benefits of exclusive breastfeeding for the first six months of life are powerful

Major takeaway
The benefits of exclusive breastfeeding for the first 6-months of life are powerful. They include improved gastrointestinal and immune function, reduced risk of infection, lower risk of developing certain conditions later in life (e.g., allergies, asthma, obesity), and strengthened maternal-child bonding.

Main explanation
Substantial evidence supports the advantages of breastfeeding over breast milk substitutes (e.g. formula) or early addition of foods. The benefits of exclusive breastfeeding are powerful for the first six months of life. Breast milk contains secretory IgA, which supplements the baby’s gastrointestinal and immune systems. Breast milk reduces the baby’s risk of developing infections and conditions such as allergies, asthma, and obesity. Moreover, breastfeeding facilitates maternal-child bonding and decreases the risk of maternal breast and ovarian cancer.
However, it should be noted that the amount of vitamin D in breast milk is typically insufficient for bone health. Exclusively breastfed infants may require vitamin D supplementation.

Contraindications to breastfeeding include maternal use of illicit substances (e.g., cocaine, heroin, cannabis). Other harmful substances, such as alcohol and tobacco, can also pass from mother to child via breast milk. Finally, maternal infections (e.g. HIV) are a contraindication to breastfeeding in high-income settings but may be appropriate in low-income settings where the cost of formula and access to clean water are serious challenges.